Re: If the universe is infinite in spatial extent, it is uncreated.

2024-09-21 Thread John Clark
On Fri, Sep 20, 2024 at 6:37 PM Alan Grayson  wrote:

*>> This entire business started by you asking what would happen at T=0 if
> the universe started running backwards and obviously, regardless of if
> space is finite or infinite, space would have to expand infinitely fast
> because at T=0 it would have a zero amount of time to expand from nothing
> to something.*
>
>
> *> Not exactly.  I figured that since the universe is expanding, we could
> run to clock backward and imagine enclosing it in a sphere,*
>

*If you're assuming that at T=0 the ENTIRE universe could be contained in a
sphere of finite size then you're assuming that space is finite, the very
thing you're trying to prove.  *


> *> say, establishing that it is finite, hence NOT flat, since flat implies
> infinite in spatial extent.*
>

*Not necessarily. In the "3-torus model" space is flat, and space is
finite, and space has no boundary, BUT it also has no edges; a 2-D analogy
of this would be a video game where if you go too far to the extreme right
you disappear and then instantly reappear on the extreme left. However
nobody knows what the large scale topology of the universe is, not even
you.  *

*Before the discovery of Dark Energy and the acceleration of the universe,
people thought if you knew the large scale topology of the universe then
you could figure out if it was infinite or finite; if it was flat then it
was infinite, if it was positively curved then it was finite, and if it was
negatively curved then it was infinite. But now things are not that simple
and there is not a clear cut relationship between shape and the finite
versus infinite question. **Even a positively curved universe could be open
and expand forever if the universe is accelerating. And flat doesn't
necessarily mean infinite.*

*> we can prove the universe is NOT flat using a purely logical argument.
> No need to do any measurements.*
>

*That's what the ancient Greeks thought, experiments are unnecessary, and
that attitude is why physics didn't advance one inch in over 1500 years. I
don't care how beautiful a philosophical argument is, if measurement
says an idea is wrong then it's wrong. *


*> I sent this analysis to a professor emeritus whose main interest is in
> cosmology who is associated with Case Western University. He replied that
> my analysis dealt only with the observable universe and that the universe
> could be infinite in spatial extent, presumably when one considers the
> unobservable part.*
>

*And that is why, as I said before, when Alan Guth wrote that long ago the
universe was just the size of a proton he meant the OBSERVABLE universe. *

*> I then realized that the unobservable part was very likely caused by
> Inflation, and therefore the entire universe would remain finite provided
> we ran the clock backward, prior to Inflation. *
>

*And as I said before, IF the universe was finite before inflation then it
was finite after it, and IF the universe was infinite before inflation it
was infinite after inflation. So inflation is irrelevant in a finite versus
infinite discussion, as should've been obvious to you because during
inflation although the universe grew by an astronomically large amount that
amount was FINITE.*

* > the concept of a created universe, one which comes into being, which is
> infinite in spatial extent, assumes a type of singularity which I believe
> is non-physical and can't be realized;*
>

*1)A belief is not a proof. *

*2) Apparently you believe a nothing to something singularity, where things
change INFINITELY (not just astronomically) fast is possible, so why is
infinite space so unbelievable?*

*3) Modern physics says a singularity occurred at T =0, but NOBODY believes
that is the last word on the subject! Everybody believes we're missing
something, but nobody knows what.   *


> *> So, the professor apparently doesn't realize that his critique of my
> original analysis implies that his claim that the universe might be
> infinite in spatial extent, contains an implicit denial it had a beginning,
> called the Big Bang.*
>

 *Alan, did it ever occur to you that a physics professor at Case Western
University who spent his life studying this subject MIGHT know more about
it than you do? *

*> I haven't written him again to relieve him of his apparent
> misconception, though I might.*


*Crackpots always believe they know more about a subject than the experts
do, and to be fair sometimes they actually do and calling them a crackpot
is a libel, but for every Galileo there are 6.02*10^23 crackpots.   *

*> I did write Alan Guth about a week ago, asking if he assumed the entire
> universe, or just the observable part existed, when Inflation began, at
> around 10^-35 seconds after the Big Bang, when the universe was around the
> size of a proton, or possibly smaller. So far he hasn't replied. *
>

*Gee I wonder why. *
 John K ClarkSee what's on my new list at  Extropolis

wyg




> 

Re: If the universe is infinite in spatial extent, it is uncreated.

2024-09-20 Thread John Clark
On Fri, Sep 20, 2024 at 11:09 AM Alan Grayson 
wrote:

*> Proof by Contradiction: If the universe is infinite in spatial extent,
> and came into being, that would be a type of singularity where it would
> have to instantaneously expand infinitely in spatial extent.*


*This entire business started by you asking what would happen at T=0 if the
universe started running backwards and obviously, regardless of if space is
finite or infinite, space would have to expand infinitely fast because at
T=0 it would have a zero amount of time to expand from nothing to
something. And yes that is a singularity however in physics, unlike pure
mathematics, when you run into a singularity what that is really telling
you is that there is some unknown physics going on that you don't
understand, or don't understand well enough. Everybody knows something is
wrong but nobody knows what. *

*By the way when people, like me, say that because of AI we're heading
towards a Singularity they are using poetic license, things in general and
society in particular won't really be changing infinitely fast, just faster
than the human meat brain can comprehend. *

John K ClarkSee what's on my new list at  Extropolis

e4b


>

-- 
You received this message because you are subscribed to the Google Groups 
"Everything List" group.
To unsubscribe from this group and stop receiving emails from it, send an email 
to everything-list+unsubscr...@googlegroups.com.
To view this discussion on the web visit 
https://groups.google.com/d/msgid/everything-list/CAJPayv1gH8TXPkJ4D21O%2BG6BdTQMJrVX%2BCEXJZFmGFnEw6uH%3DQ%40mail.gmail.com.


Three Mile Island nuclear plant​ will reopen to power Microsoft AI

2024-09-20 Thread John Clark
*Three Mile Island nuclear plant will reopen to power Microsoft AI*


 John K ClarkSee what's on my new list at  Extropolis

i3i

-- 
You received this message because you are subscribed to the Google Groups 
"Everything List" group.
To unsubscribe from this group and stop receiving emails from it, send an email 
to everything-list+unsubscr...@googlegroups.com.
To view this discussion on the web visit 
https://groups.google.com/d/msgid/everything-list/CAJPayv1VFHODdPL7t5tU7K3bFMJ_%3DK%2BKBcE%2BatUjiCKqo90-BQ%40mail.gmail.com.


Re: Trump unfit to serve say 111 former GOP officials and they endorse Harris

2024-09-19 Thread John Clark
On Wed, Sep 18, 2024 at 8:54 PM Brent Meeker  wrote:

* > But no office-holding Republicans.*
>

*Yes, I noticed that too.** And that is why, although I have been a
Republican for most of my life, I will never vote for a Republican again,
not even for dogcatcher. *

 John K ClarkSee what's on my new list at  Extropolis

ddg

>
>

-- 
You received this message because you are subscribed to the Google Groups 
"Everything List" group.
To unsubscribe from this group and stop receiving emails from it, send an email 
to everything-list+unsubscr...@googlegroups.com.
To view this discussion on the web visit 
https://groups.google.com/d/msgid/everything-list/CAJPayv3A47sEryb_VJjOTy%3DGdxFnJHs7FMCJiUfnOf8wBi87Ag%40mail.gmail.com.


New AI has an IQ of 120

2024-09-18 Thread John Clark
*This is why OpenAI o1 preview (alias strawberry, alias Q*)  is so
exciting, previous AIs have topped out at about IQ 90, but this one has a
120 IQ. Last November, when this thing was developed, it got everybody so
hot and bothered that they fired Sam Altman as head of OpenAI, and then
quickly hired him back. I wonder what they've been doing undercover at
OpenAI's skunk works for the last 10 months.  *


[image: 908B34C4-BB0E-4258-A458-E9F087B90BAA_1_105_c.jpeg]


John K ClarkSee what's on my new list at  Extropolis

2oI

-- 
You received this message because you are subscribed to the Google Groups 
"Everything List" group.
To unsubscribe from this group and stop receiving emails from it, send an email 
to everything-list+unsubscr...@googlegroups.com.
To view this discussion on the web visit 
https://groups.google.com/d/msgid/everything-list/CAJPayv2mAq0-XrOQS-uwTtOWfOU%3DH7Ai%3D7O122LsJ_o_r4Q6VQ%40mail.gmail.com.


Re: Flat universe implies no Big Bang and Singularity at T = 0

2024-09-18 Thread John Clark
On Wed, Sep 18, 2024 at 8:12 AM Alan Grayson  wrote:

>
>
> On Wednesday, September 18, 2024 at 5:40:42 AM UTC-6 John Clark wrote:
>
> On Wed, Sep 18, 2024 at 1:16 AM Alan Grayson  wrote:
>
> *I'll get back to you on this. I was thinking, as x increases positively
> or negatively, the y values (angles) repeat multiple times, making the
> function many-to-one. In this case, we're mapping all the real numbers, to
> a subset of the y-axis. Am I mistaken? AG *
>
>
> *Arctan(1) = the angle whose tangent = 1. Isn't this angle 90 deg or
> pi/2?  So your plot seems wrong, but it's what is on the Internet. AG *
>
>
> *That's wrong. Arctan(1) = pi/4, which is what the plot indicates. But I
> still think the plot keeps repeating as x increases or decreases. AG*
>
> [image: image.png]
>
>
> *1) **The range of the Arctangent function is the interval (-π/2,π/2) and
> its range is all the real numbers.*
>
> *2) By dividing by π,  the range scales to (-1/2, 1/2).*
>
> *3) Adding 1/2 shifts the range to (0,1) *
>
> *4) Thus for every real number x there is a unique number y between zero
> and one that corresponds to it, and that number is Y=1/2 + 1/π Arctan(x) .
> As I said before, the domain is all the real numbers and the range is (0,1)*
>
>
>
> *> Yes, but initially you were seeking a 1-1 function, but this one is
> many-to-one. AG *
>
>  FOR DARWIN'S SAKE! I GIVE UP!

 John K ClarkSee what's on my new list at  Extropolis
<https://groups.google.com/g/extropolis>
ugi

-- 
You received this message because you are subscribed to the Google Groups 
"Everything List" group.
To unsubscribe from this group and stop receiving emails from it, send an email 
to everything-list+unsubscr...@googlegroups.com.
To view this discussion on the web visit 
https://groups.google.com/d/msgid/everything-list/CAJPayv18HvVkAeDx1yvE1UM_K%2B4EvhqBDcXoA96V-h%3DpctxVuw%40mail.gmail.com.


Re: Radius of the Observable universe

2024-09-18 Thread John Clark
On Wed, Sep 18, 2024 at 2:01 AM Alan Grayson  wrote:

*>  I can't see how anything can recede at velocity > c and remain within
> our Observable Universe. *


*We can observe a very distant galaxy even though it is now moving away
from us faster than the speed of light because we are not observing the
galaxy as it is now, we are observing it as it was 13 billion years ago;
and back then it was NOT moving away from us faster than the speed of
light. Thus even though we can see the galaxy we could NEVER travel to it,
not even if we could move at the speed of light, not even in an infinite
number of years. You can object to the definition of "observable universe"
if you want to but remember we can NOT observe ANYTHING as it is now. It
takes a finite amount of time for light to go from the tip of your nose to
your eye, so even that observation is in the past.*

*Our observational horizon is shrinking, in about 1 trillion years we will
not be able to see any galaxies except those in our local group, and they
would probably all have merged into a single large globular galaxy by
then. So if there are any astronomers around in 1 trillion years they will
incorrectly conclude what astronomers in the early 20th century concluded,
the entire universe consists of just one galaxy surrounded by an infinity
of nothingness. That is to say surrounded by an infinite boundless
homogeneity. *

 John K ClarkSee what's on my new list at  Extropolis

ius

-- 
You received this message because you are subscribed to the Google Groups 
"Everything List" group.
To unsubscribe from this group and stop receiving emails from it, send an email 
to everything-list+unsubscr...@googlegroups.com.
To view this discussion on the web visit 
https://groups.google.com/d/msgid/everything-list/CAJPayv1AHD9TU1MNYYqCaoC9meimaamCnCh3zZE%2BpeXXu7w6%3DQ%40mail.gmail.com.


Re: Flat universe implies no Big Bang and Singularity at T = 0

2024-09-18 Thread John Clark
On Wed, Sep 18, 2024 at 1:16 AM Alan Grayson  wrote:

*I'll get back to you on this. I was thinking, as x increases positively or
> negatively, the y values (angles) repeat multiple times, making the
> function many-to-one. In this case, we're mapping all the real numbers, to
> a subset of the y-axis. Am I mistaken? AG *
>
>
> *Arctan(1) = the angle whose tangent = 1. Isn't this angle 90 deg or
> pi/2?  So your plot seems wrong, but it's what is on the Internet. AG *
>
>
> *That's wrong. Arctan(1) = pi/4, which is what the plot indicates. But I
> still think the plot keeps repeating as x increases or decreases. AG*
>
> [image: image.png]
>

*1) **The range of the Arctangent function is the interval (-π/2,π/2) and
its range is all the real numbers.*

*2) By dividing by π,  the range scales to (-1/2, 1/2).*

*3) Adding 1/2 shifts the range to (0,1) *

*4) Thus for every real number x there is a unique number y between zero
and one that corresponds to it, and that number is Y=1/2 + 1/π Arctan(x) .
As I said before, the domain is all the real numbers and the range is (0,1)*

 John K ClarkSee what's on my new list at  Extropolis

rdi

-- 
You received this message because you are subscribed to the Google Groups 
"Everything List" group.
To unsubscribe from this group and stop receiving emails from it, send an email 
to everything-list+unsubscr...@googlegroups.com.
To view this discussion on the web visit 
https://groups.google.com/d/msgid/everything-list/CAJPayv339pp-1uoRoCVfWVjHUy3CCf6cCt2VsxOB3vSFxweEUg%40mail.gmail.com.


AI chatbot got conspiracy theorists to question their convictions

2024-09-17 Thread John Clark
*AI chatbot got conspiracy theorists to question their convictions*


John K ClarkSee what's on my new list at  Extropolis

cte

-- 
You received this message because you are subscribed to the Google Groups 
"Everything List" group.
To unsubscribe from this group and stop receiving emails from it, send an email 
to everything-list+unsubscr...@googlegroups.com.
To view this discussion on the web visit 
https://groups.google.com/d/msgid/everything-list/CAJPayv0-S0TSHGNVg%3DxsLHfCif3%3DrWpGHmeZfCqiGCM6tdF_CQ%40mail.gmail.com.


Re: Radius of the Observable universe

2024-09-17 Thread John Clark
On Mon, Sep 16, 2024 at 7:31 PM Alan Grayson  wrote:

> *when we approach the observable event horizon, the spatial expansion is
> still increasing less than c,*
>

*Yes. Obviously you will not be able to see any stars receding from you
faster than c. *

> *so I don't see how the observational event horizon is 46 BLY. AG *
>

*If you are 5 billion light years away from me then in one direction you
will be able to see stars that I cannot see, but in the opposite direction
I will be able to see stars that you cannot see. And If you see the light
from a star that has travel for 13.8 billion years before entering your
telescope and then you get into a spaceship that travels at 99.999% the
speed of light for 13.8 billion years, you will still be nowhere near that
star, and if you travel for 46 billion years you will STILL be nowhere near
that star, although you will be able to see many stars that I cannot see,
but you will not be able to see any of the stars that I, who has remained
on the Earth, can currently see.  *

*> the observable universe, space is expanding at a rate less than c.
> Correct? So the 46 BLY distance doesn't seem right. *


*Even if you forget about General Relativity, and even if you forget that
the universe is accelerating, we've known for a century that the universe
is expanding, so if you're looking at a star as it was 13.8 billion years
ago then even Newton would say that by now that star is much further away
than 13.8 billion light years.  *

 John K ClarkSee what's on my new list at  Extropolis

mfa



>

-- 
You received this message because you are subscribed to the Google Groups 
"Everything List" group.
To unsubscribe from this group and stop receiving emails from it, send an email 
to everything-list+unsubscr...@googlegroups.com.
To view this discussion on the web visit 
https://groups.google.com/d/msgid/everything-list/CAJPayv3r8j7OHnV%2BUrVG87aQuwp9mDM8cwSfeQoekjxpJo%2BVLA%40mail.gmail.com.


Re: Brain aged more slowly in monkeys given a cheap diabetes drug

2024-09-17 Thread John Clark
On Tue, Sep 17, 2024 at 1:20 AM Brent Meeker  wrote:

> The link goes to a Nature article: "Dimensional crossover in a quantum
> gas of light."
> at  https://www.nature.com/articles/s41567-024-02641-7
>

*Yeah I was reading both articles and sent the wrong link. Sorry. This is
the correct link. *

*The brain aged more slowly in monkeys given a cheap diabetes drug*


John K Clark



>
>

-- 
You received this message because you are subscribed to the Google Groups 
"Everything List" group.
To unsubscribe from this group and stop receiving emails from it, send an email 
to everything-list+unsubscr...@googlegroups.com.
To view this discussion on the web visit 
https://groups.google.com/d/msgid/everything-list/CAJPayv3b7keWF8sX%3D2NkUjknX%2BBChWe9yxyxbJfTABnaZRNaNA%40mail.gmail.com.


Re: Flat universe implies no Big Bang and Singularity at T = 0

2024-09-17 Thread John Clark
On Mon, Sep 16, 2024 at 11:46 PM Alan Grayson 
wrote:

> How would you map (0,1) 1-1 onto the real numbers?
>
>
> *F(x)=1/2 + 1/π Arctan(x) . The domain is all the real numbers and the
> range is (0.1)*
>
>
> *> This map isn't 1-1. Many x's correspond to the same point in (0,1). AG *
>

*This is a graph of the Arctan function. Show me many X's, or even one
X, that corresponds to the same point in y.*

[image: image.png]


*>> If a set is infinitely large then there is a proper subset of that set
> that can be put into a one to one correspondence with the entire set, in
> fact that is the mathematical definition of "infinity".*
>
>
> *> That's circular. AG *
>
>
> *>I deleted the post with the circular comment. Why are you responding to
> it? AG*
>

*I responded to your email. Apparently you thought you knew a way to delete
an email that was already on my computer or delete my memory of reading
that email. Neither worked.   *

*>  an infinite universe cannot be created. If one exists, it is
> eternal. The reason is because the creation would require something
> non-physical; infinite spatial expansion instantaneously. So no BB for a
> spatially infinite universe.*
>


*I will now quote somebody named Alan Grayson "You keep doing the same
thing; asserting a result without proving it " *

John K ClarkSee what's on my new list at  Extropolis

agd

-- 
You received this message because you are subscribed to the Google Groups 
"Everything List" group.
To unsubscribe from this group and stop receiving emails from it, send an email 
to everything-list+unsubscr...@googlegroups.com.
To view this discussion on the web visit 
https://groups.google.com/d/msgid/everything-list/CAJPayv2vYQPnyaTL%2BcPOeWbHShRT8qWcH0w2wMDx-8ErBt_NNg%40mail.gmail.com.


Re: Flat universe implies no Big Bang and Singularity at T = 0

2024-09-16 Thread John Clark
On Mon, Sep 16, 2024 at 9:36 PM Alan Grayson  wrote:

> How would you map (0,1) 1-1 onto the real numbers?
>

*F(x)=1/2 + 1/π Arctan(x) . The domain is all the real numbers and the
range is (0.1)*

*> **Both sets have the same cardinality.*
>

* And a short line in a long line have the same cardinality, and there is a
very simple geometric proof showing that they can be put into a one to one
correspondence.  Just draw a triangle with a line from the apex running
through the short line to the long line at the base of the triangle *

*> If a set is infinitely large then there is a proper subset of that set
> that can be put into a one to one correspondence with the entire set, in
> fact that is the mathematical definition of "infinity".*


> *> That's circular. AG *



*Alan, if you open any book on set theory you will find that the formal
mathematical definition of infinity is that something is infinite if and
only if a proper subset of it can be put into a one to one correspondence
with the entire thing. *

John K ClarkSee what's on my new list at  Extropolis

ioi

-- 
You received this message because you are subscribed to the Google Groups 
"Everything List" group.
To unsubscribe from this group and stop receiving emails from it, send an email 
to everything-list+unsubscr...@googlegroups.com.
To view this discussion on the web visit 
https://groups.google.com/d/msgid/everything-list/CAJPayv2QYCGruQvWUcUNnc99nv24iEAhCtbb1AXdNoC77x3S-w%40mail.gmail.com.


Re: Brain aged more slowly in monkeys given a cheap diabetes drug

2024-09-16 Thread John Clark
*Sorry I gave a bad link. Let me try that again: *

*The brain aged more slowly in monkeys given a cheap diabetes drug*
<https://www.nature.com/articles/d41586-024-02938-w>

John K Clark

On Mon, Sep 16, 2024 at 4:34 PM John Clark  wrote:

> *Brain aged more slowly in monkeys given a cheap diabetes drug*
> <https://www.nature.com/articles/s41567-024-02641-7>
>
> John K ClarkSee what's on my new list at  Extropolis
> <https://groups.google.com/g/extropolis>
> qez
>
>
>

-- 
You received this message because you are subscribed to the Google Groups 
"Everything List" group.
To unsubscribe from this group and stop receiving emails from it, send an email 
to everything-list+unsubscr...@googlegroups.com.
To view this discussion on the web visit 
https://groups.google.com/d/msgid/everything-list/CAJPayv2f3bEPTLv%3DV%2Bq6LymYA4Sr%3DNtH7a0fRr2B9h2WhyMDpA%40mail.gmail.com.


Re: Flat universe implies no Big Bang and Singularity at T = 0

2024-09-16 Thread John Clark
On Mon, Sep 16, 2024 at 4:51 PM Alan Grayson  wrote:

>  *an infinite universe cannot expand*


*NO! In an infinite universe the distance between every particle in it and
every other particle in it can still increase indefinitely. If a set is
infinitely large then there is a proper subset of that set that can be put
into a one to one correspondence with the entire set, in fact that is the
mathematical definition of "infinity". *

 John K ClarkSee what's on my new list at  Extropolis

imd

-- 
You received this message because you are subscribed to the Google Groups 
"Everything List" group.
To unsubscribe from this group and stop receiving emails from it, send an email 
to everything-list+unsubscr...@googlegroups.com.
To view this discussion on the web visit 
https://groups.google.com/d/msgid/everything-list/CAJPayv1%2BOS7grhrf7g8pRU7dwyojtLJXVeApJfDDd00bVvE_eQ%40mail.gmail.com.


Brain aged more slowly in monkeys given a cheap diabetes drug

2024-09-16 Thread John Clark
*Brain aged more slowly in monkeys given a cheap diabetes drug*


John K ClarkSee what's on my new list at  Extropolis

qez

-- 
You received this message because you are subscribed to the Google Groups 
"Everything List" group.
To unsubscribe from this group and stop receiving emails from it, send an email 
to everything-list+unsubscr...@googlegroups.com.
To view this discussion on the web visit 
https://groups.google.com/d/msgid/everything-list/CAJPayv3_D5tNZ1p0ZQEtqzdF5vb4hwg5fF41NuZn%2BOZ0_2n9iA%40mail.gmail.com.


Re: Radius of the Observable universe

2024-09-16 Thread John Clark
On Mon, Sep 16, 2024 at 10:40 AM Alan Grayson 
wrote:

*>> It has taken light from a star (or more likely from the CMB) 13.8
> billion years to reach us but during those 13.8 billion years the star has
> not remained stationary relative to us, it has been accelerating away. In
> fact telescopic observation tells us that 9 billion years ago, when Dark
> Energy became more dominant than Dark Matter (plus regular matter), the
> acceleration has been accelerating. This *MIGHT* be because as the universe
> expands Dark Matter (plus regular matter) becomes more dilute but Dark
> Energy does not become diluted because it is an intrinsic part of space
> itself, so the more space you have the more Dark Energy you have.*
>
>
> *> If an object is receding for 13.8 BY, and the universe is expanding
> during that time, doesn't that imply a recession velocity faster than c,
> for the object to be on our observational horizon of 46 BLY? AG *
>

*Yes and that is a clear violation of Special Relativity, however that was
not Einstein's last word on the subject, that came 10 years later with
General Relativity.  Einstein still says matter, energy and information
cannot travel through space faster than light, BUT space itself is free to
expand at any speed, including much faster than light.*

 John K ClarkSee what's on my new list at  Extropolis

*mfl*

-- 
You received this message because you are subscribed to the Google Groups 
"Everything List" group.
To unsubscribe from this group and stop receiving emails from it, send an email 
to everything-list+unsubscr...@googlegroups.com.
To view this discussion on the web visit 
https://groups.google.com/d/msgid/everything-list/CAJPayv2qUmn1UgR9U0Tg4YA4J_rGZ1EbGErVhJzSfqCX6VtAnw%40mail.gmail.com.


Re: Flat universe implies no Big Bang and Singularity at T = 0

2024-09-16 Thread John Clark
On Mon, Sep 16, 2024 at 10:54 AM Alan Grayson 
wrote:

*> At t = 0, what an infinite universe in spatial extent implies; namely,
> no big bang, since that would require creating infinite spatial extent
> instantaneously*


*Neither Quantum Mechanics nor General Relativity can explain how something
with infinite spatial extent could instantaneously come into existence at
t=0, but they can't explain how something with finite spatial extent could
do so either.  If we can ever find a way to stop those two theories from
fighting each other, maybe we could figure it out.*


*> Another way to look at it is this; if the universe was finite in spatial
> extent when the BB occurred, it will always remain finite, but if it was
> infinite in spatial extent when the BB "occurred", it was always infinite*


*As I said in my previous post, if it's infinite now then it was infinite
at the time of the Big Bang, and if it was finite then it's finite now.*

*> and the BB didn't occur.*


*That does not compute.  *
 John K ClarkSee what's on my new list at  Extropolis

stn

-- 
You received this message because you are subscribed to the Google Groups 
"Everything List" group.
To unsubscribe from this group and stop receiving emails from it, send an email 
to everything-list+unsubscr...@googlegroups.com.
To view this discussion on the web visit 
https://groups.google.com/d/msgid/everything-list/CAJPayv38YR-ta7WvfU%2B%2BpCeOKRVHRehU_-EsDPca9sMb_-Gjrw%40mail.gmail.com.


Re: Radius of the Observable universe

2024-09-16 Thread John Clark
On Mon, Sep 16, 2024 at 12:53 AM Alan Grayson 
wrote:

*> It's claimed to be 46 billion LY, but its age is only measured as 13.8
> billion years. What I find puzzling about these numbers is that it seems
> this would imply the rate of expansion must have been greater than c during
> its lifetime.*



*No. It has taken light from a star (or more likely from the CMB) 13.8
billion years to reach us but during those 13.8 billion years the star has
not remained stationary relative to us, it has been accelerating away. In
fact telescopic observation tells us that 9 billion years ago, when Dark
Energy became more dominant than Dark Matter (plus regular matter), the
acceleration has been accelerating. This *MIGHT* be because as the universe
expands Dark Matter (plus regular matter) becomes more dilute but Dark
Energy does not become diluted because it is an intrinsic part of space
itself, so the more space you have the more Dark Energy you have.*

 John K ClarkSee what's on my new list at  Extropolis

mde




>

-- 
You received this message because you are subscribed to the Google Groups 
"Everything List" group.
To unsubscribe from this group and stop receiving emails from it, send an email 
to everything-list+unsubscr...@googlegroups.com.
To view this discussion on the web visit 
https://groups.google.com/d/msgid/everything-list/CAJPayv3fKVtciHxFUmKhcm1WZ8DHATeMWFO1fdAUB6n4Taz_dQ%40mail.gmail.com.


Re: Flat universe implies no Big Bang and Singularity at T = 0

2024-09-16 Thread John Clark
On Mon, Sep 16, 2024 at 6:58 AM Alan Grayson  wrote:

*> If Flat implies infinite in spatial extent, it can't be realized
> instantaneously at T = 0 (a type of singularity IMO). AG*



*Regardless of if the universe is infinite or finite or flat or curved, we
know that our current physics cannot tell us what things were like at T=0,
and we will never know until we resolve the discrepancy between our best
theory of gravity, General Relativity, and our best theory of the other
three forces of nature, Quantum Mechanics. Both theories have been tested
with incredibly precise experiments and both theories have passed with
flying colors. Physicists have been trying for nearly a century to find a
way to get those two very successful theories to play nice together, but so
far no luck.*

 John K ClarkSee what's on my new list at  Extropolis

cfl

-- 
You received this message because you are subscribed to the Google Groups 
"Everything List" group.
To unsubscribe from this group and stop receiving emails from it, send an email 
to everything-list+unsubscr...@googlegroups.com.
To view this discussion on the web visit 
https://groups.google.com/d/msgid/everything-list/CAJPayv1xNn6NafTxd7pP%3DrWrwonHo-g9up8xNS-Jn-B3wO5iFA%40mail.gmail.com.


Recent advances in quantum computing

2024-09-15 Thread John Clark
*Microsoft  announced they have entangled 12 logical qubits, the largest
number ever entangled, on a 56-physical-qubit trapped-ion processor. They
report a 0.2% error rate, which is more than 11 times better than they
would’ve gotten without using quantum error-correction.*

*Demonstration of quantum computation and error correction with a tesseract
code* 

*A**lso, researchers from Google report that error correction increases the
length of time a qubit can store information in memory.*

*Quantum error correction below the surface code threshold*


*I was reading Scott Aaronson's quantum computing blog about these recent
developments, and I was interested in the following quote because in the
past Aaronson has often been skeptical about so-called breakthroughs in
quantum computing, but not this time: *

*"One experimental milestone after another that people talked about since
the 90s is finally being achieved, to the point where it’s become hard to
keep up with it all. Let me end by sticking my neck out. If hardware
progress continues at the rate we’ve seen for the past year or two, then I
find it hard to understand why we won’t have useful fault-tolerant QCs
within the next decade. (And now to retreat my neck a bit: the “if” clause
in that sentence is important and non-removable!)" *

*And I was amused when Aaronson made this aside on a completely different
subject: *

*"I’d been wavering—should I vote for the terrifying lunatic, ranting about
trans criminal illegal aliens cooking cat meat, or for the nice woman
constantly making faces as though the lunatic was completely cracking her
up? But when the woman explicitly came out in favor of AI and quantum
computing research … that really sealed the deal for me."*

*Quantum fault-tolerance milestones dropping like atoms*


 John K ClarkSee what's on my new list at  Extropolis

std

-- 
You received this message because you are subscribed to the Google Groups 
"Everything List" group.
To unsubscribe from this group and stop receiving emails from it, send an email 
to everything-list+unsubscr...@googlegroups.com.
To view this discussion on the web visit 
https://groups.google.com/d/msgid/everything-list/CAJPayv1vXAEQvMpVtSay5egYP3ZJC9Dbn6YLbso%2BSxVjQfet_A%40mail.gmail.com.


Re: Length contraction in Special Relativity

2024-09-15 Thread John Clark
On Sun, Sep 15, 2024 at 3:26 AM Alan Grayson  wrote:

 > *I prefer approximately spherical compared to flat because as we go
> backward in time, we can enclose the universe in a sphere, implying it
> is finite in spatial extent (not infinite).*


*There is no such implication. If the universe is a 3-sphere then it could
be finite or infinite. A  3-sphere is a compact, connected, 3-dimensional
manifold without boundary embedded in 4-space, any loop on a 3-sphere can
be continuously shrunk to a point without leaving the 3-sphere.*

* > It then occurred to me that the Unobservable universe was plausibly
> created during Inflation, *


*NO. The rate of expansion during inflation was mind blowingly gargantuan,
but it was finite. If the universe was infinite before inflation then it
was infinite after it, and if it was finite before inflation then it was
finite after it.  *

*> to Alan Guth. I asked him, when he assumes the universe was around the
> size of a proton when Inflation began, was he referring only to the
> Observable universe,*


*I know for a fact Guth was referring to the observable universe because
he's a good enough physicist to know that a proton and the observable
universe have one thing in common, both of them are finite in size. And no
physical process can turn a finite thing into an infinite thing. *

* > or both hypothetic parts, Observable and Unobservable. *


*If the universe has any curvature at the largest possible scale it is
unobservable, and if you insist that postulating that something you cannot
observe and will never be able to observe nevertheless exists is
unscientific, then you would have to conclude that the pope was right and
Galileo was wrong because the Earth really is the center of the universe.
Do you really want to insist on that?  *
  John K ClarkSee what's on my new list at  Extropolis


uss



>

-- 
You received this message because you are subscribed to the Google Groups 
"Everything List" group.
To unsubscribe from this group and stop receiving emails from it, send an email 
to everything-list+unsubscr...@googlegroups.com.
To view this discussion on the web visit 
https://groups.google.com/d/msgid/everything-list/CAJPayv3hPmd-V4p9a93iN_Q%3D4Q4NDhsdxcOkvEGGghMQ0w0YVw%40mail.gmail.com.


Can ChatGPT o1-preview Solve PhD-level Physics Textbook Problems?

2024-09-14 Thread John Clark
*This guy was skeptical about open AI's claim so he got the toughest PHD
level textbook that he could find and asked GPT o1-preview (A.K.A. Q*
A.K.A. strawberry)  3 problems from it, the sort of problems a PhD student
would take a week to solve, and this AI ended up solving all three in less
than five minutes.  And this wasn't even the full program, it was just a
previewed cut down version of the real thing!*


*Can ChatGPT o1-preview Solve PhD-level Physics Textbook Problems?*


John K ClarkSee what's on my new list at  Extropolis

evw

-- 
You received this message because you are subscribed to the Google Groups 
"Everything List" group.
To unsubscribe from this group and stop receiving emails from it, send an email 
to everything-list+unsubscr...@googlegroups.com.
To view this discussion on the web visit 
https://groups.google.com/d/msgid/everything-list/CAJPayv0A_UscCkdwPW9KOfg3JaKOpPgvzvnqP0XUxneKVjWFfg%40mail.gmail.com.


Re: Amoeba's Secret openly available under CC-BY license

2024-09-14 Thread John Clark
On Fri, Sep 13, 2024 at 5:51 PM Brent Meeker  wrote:

* >> physics is the language of mathematics, it's very good at describing
>> things. An equation can describe how system X, that is made of mass/energy,
>> can in an interval of time change into something different, system Y.  But
>> a language by itself can't do anything because neither mathematics nor the
>> English language can change with time unless there is a brain made of atoms
>> to think about them. *
>
>
> *> Language is representation. *
>

* Yes, as I said mathematics is "very good at describing things".*


*> **Its "energy" can't do work either.  Its "distance" isn't far away
> whatever you think about it. *


*Work is force over a "distance", but for the very concept of "distance" to
be meaningful it must have four properties: *

*1) The distance between X and Y must be greater than or equal to zero for
all X and Y.*
*2) The distance between X and Y equals zero if and only if X=Y.*
*3) The distance between X and Y must be the same as the  distance
between Y and X.*
*4) The distance between X and Z must be less than or equal to the distance
between X and Y plus the distance between Y and Z, for all X, Y, and Z.*

*The real numbers and the complex numbers all have the above four
properties so they could be used to measure "distance" BUT it turns out
there is another fundamentally different type of number that also has those
four properties called P-adic numbers.  For any prime number p, is a p-adic
number system and in those systems the size of a number does not depend on
how far it is from zero but by how the visible that number is by powers of
P. It can lead to some unintuitive results, in the 5-adic system, 25 is
smaller than 5.*

*But if both the Real Numbers and the P-adic numbers are equally consistent
why don't we start teaching first graders about P-adic numbers? Because the
real numbers are extremely useful in physics, a.k.a. the physical world,
but the  P-adic numbers are almost useless in physics, although they are
helpful  in pure mathematics, Andrew Wiles used them to prove Fermat's Last
Theorem,*

*> mathematics doesn't change just because you think about it.*

*Yes but that's the problem, the** very fact that mathematics doesn't
change in time is the reason it can't produce intelligence or
consciousness. Physics is needed for that.*

*>> Neither intelligence nor consciousness can exist without something
>> changing with time. *
>
>
> *>I think you're just making a philosophical position out of a tautology
> change=>time.*


*The great thing about tautologies is that all of them are 100% true.  *

*You might as well add motion=>distance and heating=>temperature.*
>

*I can imagine that an intelligent conscious being might not be changing
its position in space, and it might not be changing its temperature, but I
can't imagine an intelligent conscious being not changing its thoughts in
time. The trouble with pure mathematics is that it's timeless. *

*>> The symbols that make up mathematical equations can't change with time,
>> *
>
>
> *> They can't change with distance either, or the ink used, or motion,...
> But their meaning changes in different applications.*


*"Meaning" does not "mean" anything unless there is an intelligent
conscious being around because they are the only ones that are in the
meaning conferring business.   *

*> Modern philosophers define "reality" as a substance that actually exists
> in an external world, and they define "existence" as the state of having
> "reality". And round and round we go.That's why I say in the modern age
> it's mathematicians and physicists who are in the vanguard of the
> investigation into the fundamental nature of reality, while those who write
> "philosopher" on their tax forms when it asks about occupation are really
> in the synonym business not the philosophy business.  **And I would also
> maintain that the semiconductor industry is an existence proof that
> equations alone are not sufficient because they can't DO anything. **And
> I would also maintain that the semiconductor industry is an existence proof
> that equations alone are not sufficient because they can't DO anything.*


*> The existence of something that's not equations alone and does
> something, is not a proof that nothing is done by equations alone. It's a
> proof that at least one thing requires more than equations to be done. *
>

*It's proof that if you want something that can  DO things, anything, then
it's going to need to have Physics up its sleeve because pure mathematics
is not sufficient. *


*>> If you want to actually DO something, that is to say if you want to
>> make a change over a period of time, then you're going to need mass/energy.*
>
>
> > *Actually those are conserved.  What you need is low entropy energy. *


*Conservation is irrelevant. What you need if you want to produce
intelligence or consciousness is an arrangement of atoms that can process
information; those configurations tend to

Re: Amoeba's Secret openly available under CC-BY license

2024-09-13 Thread John Clark
On Fri, Sep 13, 2024 at 1:08 AM Bruce Kellett  wrote:


>> *>> I thought you could get the appearance of randomness from a
>> first-person perspective in MW? Has that been shown to not work?*
>>
>
> *> I don't think that works. The idea often put forward is something along
> the lines of self-locating uncertainty -- out of all the branches, which
> one am I on? But that is only apparent randomness,*
>

*True. Schrodinger's wave function is completely deterministic so if you
knew the wave function for the entire universe then nothing would be
random, but of course we don't know the universal wave function and never
will. *


> *> and to get such an idea to work, you need to be able to make a random
> choice between branches.*
>

*Choice has nothing to do with it. The one and only assumption Many Worlds
makes is that everything always follows Schrodinger's equation.  And that
means that everything consistent with that equation will happen. And you
turning right is consistent with Schrodinger so that will happen, and you
turning left is consistent with Schrodinger so that will happen, but an
electron turning into a proton is not consistent with Schrodinger so that
will never happen. If Many Worlds is correct then randomness is just
apparent, not objectively intrinsic. *


> *> GRW collapse theory: it is perhaps the only theory around at the moment
> that has an explanation of intrinsic randomness,*
>

*I think Many Worlds is probably correct but of course I can't be certain,
and I have to give credit to GRW and other intrinsic collapse theories
because they make experimentaly testable predictions. I doubt it will
happen but if their predictions turn out to be true then there is no wiggle
room, they will have proved that Many Worlds is dead wrong. However they do
not give an explanation for randomness, GRW and related theories just stick
in additional stuff, including two new constants of nature, into
Schrödinger's Wave Equation that generates randomness. And the constants
were not picked for any theoretical reason, instead it was completely ad
hoc;  if those two  constants were any larger we would have already
detected them, and if they were any smaller they would be too small to get
the job done. *

*Getting back to "choice", there are only two possibilities, you did what
you did for a reason or you did what you did for no reason. If Many Worlds
is correct then there is always a reason why you did what you did, although
you may not know what it is. If GRW is correct then some effects have no
cause. Time will tell which is right.  *

John K ClarkSee what's on my new list at  Extropolis

twt

-- 
You received this message because you are subscribed to the Google Groups 
"Everything List" group.
To unsubscribe from this group and stop receiving emails from it, send an email 
to everything-list+unsubscr...@googlegroups.com.
To view this discussion on the web visit 
https://groups.google.com/d/msgid/everything-list/CAJPayv246_skDLhPgttd1J2wcae1ebokmKw_iPsUFgu1dRMbKw%40mail.gmail.com.


Proof that Trump lost the debate

2024-09-13 Thread John Clark
*Shares of Trump Media & Technology Group, the parent company of Truth
Social, fell more than 10 percent on the day after the debate.*

 John K ClarkSee what's on my new list at  Extropolis

dlt

-- 
You received this message because you are subscribed to the Google Groups 
"Everything List" group.
To unsubscribe from this group and stop receiving emails from it, send an email 
to everything-list+unsubscr...@googlegroups.com.
To view this discussion on the web visit 
https://groups.google.com/d/msgid/everything-list/CAJPayv0r9bo0cd-xdc7sbLNwhdfg-PEh%3D%2BoBDSHVi4ah9PvFvw%40mail.gmail.com.


NYTimes.com: OpenAI Unveils New ChatGPT That Can Reason Through Math and Science

2024-09-13 Thread John Clark
Explore this gift article from The New York Times. You can read it for free
without a subscription.

OpenAI Unveils New ChatGPT That Can Reason Through Math and Science

Driven by new technology called OpenAI o1, the chatbot can test various
strategies and try to identify mistakes as it tackles complex tasks.

https://www.nytimes.com/2024/09/12/technology/openai-chatgpt-math.html?unlocked_article_code=1.KU4.ATb5.D8MlZsVVTqhU&smid=em-share

-- 
You received this message because you are subscribed to the Google Groups 
"Everything List" group.
To unsubscribe from this group and stop receiving emails from it, send an email 
to everything-list+unsubscr...@googlegroups.com.
To view this discussion on the web visit 
https://groups.google.com/d/msgid/everything-list/CAJPayv1iXZzh%3Dsuh7tVMWF2CSgSzWcnwADTBABcuHbzBjSvYQg%40mail.gmail.com.


Re: Amoeba's Secret openly available under CC-BY license

2024-09-13 Thread John Clark
On Thu, Sep 12, 2024 at 12:40 AM Liz R  wrote:

>  *physics is possibly isomorphic to some set of equations that describe
> reality, *


*I agree, and that's why physics is the language of mathematics, it's very
good at describing things. An equation can describe how system X, that is
made of mass/energy, can in an interval of time change into something
different, system Y.  But a language by itself can't do anything because
neither mathematics nor the English language can change with time unless
there is a brain made of atoms to think about them. Neither intelligence
nor consciousness can exist without something changing with time. The
symbols that make up mathematical equations can't change with time, but
carbon atoms can, so you can erase a symbol in an equation that you
penciled in and lay down a different pattern of carbon atoms that
represents a different symbol.*

*> Occam suggests that we don't actually need reality to exist, only the
> equations.*


*Modern philosophers define "reality" as a substance that actually exists
in an external world, and they define "existence" as the state of having
"reality". And round and round we go.That's why I say in the modern age
it's mathematicians and physicists who are in the vanguard of the
investigation into the fundamental nature of reality, while those who write
"philosopher" on their tax forms when it asks about occupation are really
in the synonym business not the philosophy business.  *

*And I would also maintain that the semiconductor industry is an existence
proof that equations alone are not sufficient because they can't DO
anything. If you want to actually DO something, that is to say if you want
to make a change over a period of time, then you're going to need
mass/energy.*

John K ClarkSee what's on my new list at  Extropolis

epm

-- 
You received this message because you are subscribed to the Google Groups 
"Everything List" group.
To unsubscribe from this group and stop receiving emails from it, send an email 
to everything-list+unsubscr...@googlegroups.com.
To view this discussion on the web visit 
https://groups.google.com/d/msgid/everything-list/CAJPayv3LpQEwAbde_oDr9pTVZBPfWz1p3jgU8zosvZro4yGo%3Dw%40mail.gmail.com.


By far this is the smartest AI yet

2024-09-12 Thread John Clark
*This blows every other AI out of the water when it comes to programming,
mathematics and science.  And the amazing/scary thing is that it's
scalable, the more raw compute power you throw at it the smarter it gets.
All you need is hardware.  It's called "Open AI o1" a.k.a.  strawberry
a.k.a. Q*, I think it's the most important development in the field of AI
since  the introduction of the original GPT. *

*OpenAI's New AI GPT-o1 STUNS The ENTIRE INDUSTRY Surprises Everyone!
(STRAWBERRY RELEASED!)* 

John K ClarkSee what's on my new list at  Extropolis

sqo

-- 
You received this message because you are subscribed to the Google Groups 
"Everything List" group.
To unsubscribe from this group and stop receiving emails from it, send an email 
to everything-list+unsubscr...@googlegroups.com.
To view this discussion on the web visit 
https://groups.google.com/d/msgid/everything-list/CAJPayv1L593eMfUhb%3DAg3wLRVU6s5MrSqK_t5%3D6odhfjvW%3D-7g%40mail.gmail.com.


Fwd: Contra DeBoer On Temporal Copernicanism

2024-09-12 Thread John Clark
-- Forwarded message -
From: Astral Codex Ten 
Date: Tue, Sep 10, 2024 at 1:07 AM
Subject: Contra DeBoer On Temporal Copernicanism
To: 


...
͏     ­͏     ­͏     ­͏     ­͏     ­͏     ­͏     ­͏     ­͏     ­͏     ­͏
­͏     ­͏     ­͏     ­͏     ­͏     ­͏     ­͏     ­͏     ­͏     ­͏     ­͏
  ­͏     ­͏     ­͏     ­͏     ­͏     ­͏     ­͏     ­͏     ­͏     ­͏     ­͏
    ­͏     ­͏     ­͏     ­͏     ­͏     ­͏     ­͏     ­͏     ­͏     ­͏
­͏     ­͏     ­͏     ­͏     ­͏     ­͏     ­͏     ­͏     ­͏     ­͏     ­͏
  ­͏     ­͏     ­͏     ­͏     ­͏     ­͏     ­͏     ­͏     ­͏     ­͏     ­͏
    ­͏     ­͏     ­͏     ­͏     ­͏     ­͏     ­͏     ­͏     ­͏     ­͏
­͏     ­͏     ­͏     ­͏     ­͏     ­͏     ­͏     ­͏     ­͏     ­͏     ­͏
  ­͏     ­͏     ­͏     ­͏     ­͏     ­͏     ­͏     ­͏     ­͏     ­͏     ­͏
    ­͏     ­͏     ­͏     ­͏     ­͏     ­͏     ­͏     ­͏     ­͏     ­͏
­͏     ­͏     ­͏     ­͏     ­͏     ­͏     ­͏     ­͏     ­͏     ­͏     ­͏
  ­͏     ­͏     ­͏     ­͏     ­͏     ­͏     ­͏     ­͏     ­͏     ­͏     ­͏
    ­͏     ­͏     ­͏     ­͏     ­͏     ­͏     ­͏     ­͏     ­͏     ­͏
­͏     ­͏     ­͏     ­͏     ­͏     ­͏     ­͏     ­͏     ­͏     ­͏     ­͏
  ­͏     ­͏     ­͏     ­͏     ­͏     ­͏     ­͏     ­͏     ­͏     ­͏     ­͏
    ­͏     ­͏     ­͏     ­͏     ­͏     ­͏     ­͏     ­͏     ­͏     ­͏
­͏     ­͏     ­͏     ­͏     ­͏     ­͏     ­͏     ­͏     ­͏     ­͏     ­͏
  ­͏     ­͏     ­͏     ­͏     ­͏     ­͏     ­͏     ­͏     ­͏     ­͏     ­͏
    ­͏     ­͏     ­͏     ­͏     ­͏     ­͏     ­͏     ­
Forwarded this email? Subscribe here

for more
Contra DeBoer On Temporal Copernicanism

...

Sep 10






READ IN APP



Freddie deBoer has a post on what he calls “the temporal Copernican
principle.”
He
argues we shouldn’t expect a singularity, apocalypse, or any other
out-of-distribution thing in our lif

Re: Length contraction in Special Relativity

2024-09-11 Thread John Clark
Alan Grayson  wrote:

*> For galaxies to fall out of view, they have to moving at greater than c.
> Now they aren't receding that fast. How will they start moving that fast?*
>

*What the hell?  *

*> You're applying Hubble's law without thinking what it says. Just because
> a galaxy is now receding at less than c, how will continued expansion
> increase that speed to greater than c? AG *
>

*What the hell? Quentin gave the correct answer to both of your questions: *

*> "The farther they are the faster they are receding from you, so as they
> continue to get farther away they receed faster from you till the point
> they receed faster than c and go out of your horizon". *
>

*Alan, that is something they would teach you on the very first day of an
astronomy 101 class if you hadn't already learned it in high school, which
makes your previous two questions even more bizarre. You should have at
least a little understanding of the basics of classical physics before you
start worrying about the subtleties of the metric tensor in General
Relativity. *

*> Hubble discovered that the universe was expanding faster in the past,
> than in the present. *


*No it did not. If you're talking about cosmic inflation, that is a
hypothesis that immediately after the Big Bang for about 10^-35 seconds the
universe expanded at an exponential rate; the idea seems reasonable but it
has not been proven. What we know for sure is that Hubble (the man not the
telescope) discovered in the 1920s that the universe is expanding, and
thanks to a group of astronomers in 1997 we know that for unknown reasons
the universe's expansion is accelerating. So what we know for sure is that
in the past the universe was expanding *SLOWER* than it is now.  *

*Very recently there have been some tentative indications that the rate of
change of acceleration (the official technical term for that is "jerk") of
the universe is not zero but is negative. In other words the universe
*might* be decelerating, but of course even if it turns out to be true that
doesn't mean the expansion of the universe is slowing down, although that
might happen eventually if the deceleration continues for long enough, if
it continues for long enough the universe might even start to collapse, but
nobody knows if it will because nobody knows what the hell Dark Energy is.*
  *John* K ClarkSee what's on my new list at  Extropolis

smt

-- 
You received this message because you are subscribed to the Google Groups 
"Everything List" group.
To unsubscribe from this group and stop receiving emails from it, send an email 
to everything-list+unsubscr...@googlegroups.com.
To view this discussion on the web visit 
https://groups.google.com/d/msgid/everything-list/CAJPayv2evBDWQxQ7z3VEDe0Qfw2n7Dj%3DN5NPq4Vnpqt_HHKGig%40mail.gmail.com.


Trump told 33 lies, Harris told one

2024-09-11 Thread John Clark
*From the article: *

*"Trump made at least 33 false claims. 33.  By contrast, by a preliminary
count, Vice President Harris made at least one false claim, though she
added at least a few misleading claims and a few more that lack key
context. I think a lot of Americans say, “Well, all politicians lie.”  No
major presidential candidate before Donald Trump has ever lied with this
kind of frequency. A remarkably large chunk of what he said tonight was
just not true, and this wasn’t like little exaggeration, political spin. A
lot of his false claims were untethered to reality."*

*Trump Made at Least 33 False Claims’ in Debate, While ‘Harris Made at
Least One*


 John K ClarkSee what's on my new list at  Extropolis

utr

-- 
You received this message because you are subscribed to the Google Groups 
"Everything List" group.
To unsubscribe from this group and stop receiving emails from it, send an email 
to everything-list+unsubscr...@googlegroups.com.
To view this discussion on the web visit 
https://groups.google.com/d/msgid/everything-list/CAJPayv0x7k47tCw8d6xfK4S5WD9y6_a85LKhJRvqPy58ow214Q%40mail.gmail.com.


Re: Amoeba's Secret openly available under CC-BY license

2024-09-10 Thread John Clark
On Tue, Sep 10, 2024 at 4:44 PM Brent Meeker  wrote:

*>* *Given any sequence of states you can label them so as to represent a
> computation.  So I think the physics is really incidental to the
> computation.*
>

*You need to make the labels, and making something involves a change, and a
change cannot happen without the involvement of matter and the laws of
physics.  *

  John K ClarkSee what's on my new list at  Extropolis

uwx




>

-- 
You received this message because you are subscribed to the Google Groups 
"Everything List" group.
To unsubscribe from this group and stop receiving emails from it, send an email 
to everything-list+unsubscr...@googlegroups.com.
To view this discussion on the web visit 
https://groups.google.com/d/msgid/everything-list/CAJPayv2m4k%3DZYCiBch-kuSZyXeohy%3DAkbBs6szOBJNavEmZ3RA%40mail.gmail.com.


Re: Length contraction in Special Relativity

2024-09-10 Thread John Clark
On Tue, Sep 10, 2024 at 3:57 PM Alan Grayson  wrote:


*>> Even if you ignore Dark Energy and postulate that the Hubble constant
> really is constant, every object a megaparsec away (3.26 million
> light-years) is moving away from us at about 70 kilometers per second. So
> if you try to look at objects a sufficiently large number of megaparsec
> away you will fail to find any because they are moving away from us faster
> than the speed of light.*
>
>
> >* That was in the past. At present, the universe is expanding at about
> 70 km/sec.*
>

*Galaxies are receding from the Earth at 70 km/sec for EACH megaparsec
distant from Earth they are. The further from Earth they are, the faster
they are moving away from us, so if they are far enough away they will be
moving faster than the speed of light away from us. *

*> You're assuming the universe today is infinite,*
>

*NO! I said IF the entire universe is infinite today then it was always
infinite, and IF it was finite 10^-35 seconds after the Big Bang then it's
still finite today. I also said nobody knows if the entire universe is
infinite or finite. *


> *>* *Hubble's law applies to the past, not to the future,*
>

*What the hell?!  *
* John* K ClarkSee what's on my new list at  Extropolis


hwt


>

-- 
You received this message because you are subscribed to the Google Groups 
"Everything List" group.
To unsubscribe from this group and stop receiving emails from it, send an email 
to everything-list+unsubscr...@googlegroups.com.
To view this discussion on the web visit 
https://groups.google.com/d/msgid/everything-list/CAJPayv3Aevm7aLog7kYHcX70c%3DM3uMpu0NzsRJBYB3hhTbEwjw%40mail.gmail.com.


Re: Length contraction in Special Relativity

2024-09-10 Thread John Clark
On Tue, Sep 10, 2024 at 9:43 AM Alan Grayson  wrote:

*>> even if the rate of expansion stopped increasing it would still be true
> that some galaxies we can see today we won't be able to see tomorrow. *
>
>
> *> But you haven't explained WHY that is the case. AG *
>

*Even if you ignore Dark Energy and postulate that the Hubble constant
really is constant, every object a megaparsec away (3.26 million
light-years) is moving away from us at about 70 kilometers per second. So
if you try to look at objects a sufficiently large number of megaparsec
away you will fail to find any because they are moving away from us faster
than the speed of light.*


*>> If the entire universe, observable plus unobservable, is infinite today
> then it must've been infinite even before inflation started, it must've
> been infinite from the first Planck Time Instant of its creation. I think
> most cosmologists would say that at the largest level space is curved into
> some unknown shape but inflation has flattened it out so much the curvature
> is too small to ever be detected.  *
>
>
> *> You're assuming what I'd like to see argued.*
>

*In the above what specifically do you think I'm assuming?  *


> *> Guth says Inflation started at t = 10^-35 seconds and the universe was
> around the size of a proton.*
>

*I'm certain Guth meant the size of the OBSERVABLE universe was the size of
a proton at 10^-35 seconds. But if the entire universe, observable plus
unobservable, is infinite now then it must've been infinite then. Nobody
knows if the universe is finite or infinite.   *

*> I think the huge expansion created the unobserved region since space
> must have expanded greater than the speed of light. AG *
>


*During inflation the universe expanded VASTLY faster than the speed of
light, but even without inflation we still wouldn't be able to see things
further than 13.8 billion light years away for obvious reasons.   *

 John K ClarkSee what's on my new list at  Extropolis

RRO

-- 
You received this message because you are subscribed to the Google Groups 
"Everything List" group.
To unsubscribe from this group and stop receiving emails from it, send an email 
to everything-list+unsubscr...@googlegroups.com.
To view this discussion on the web visit 
https://groups.google.com/d/msgid/everything-list/CAJPayv2sa_A944MsJEXyt_9q84ESW3rE_JCi07TmWC%3Ds9VVBoQ%40mail.gmail.com.


NYTimes.com: Donald Trump on the Dollar, in His Own Words

2024-09-10 Thread John Clark
Explore this gift article from The New York Times. You can read it for free
without a subscription.

Donald Trump on the Dollar, in His Own Words

Another day, another bizarre tariff proposal.

https://www.nytimes.com/2024/09/09/opinion/trump-tariffs-reserve-currency.html?unlocked_article_code=1.Jk4.Ppdi.rhahiLNljoJe&smid=em-share

-- 
You received this message because you are subscribed to the Google Groups 
"Everything List" group.
To unsubscribe from this group and stop receiving emails from it, send an email 
to everything-list+unsubscr...@googlegroups.com.
To view this discussion on the web visit 
https://groups.google.com/d/msgid/everything-list/CAJPayv1g6OnSW7XJ7tCqvgNFqK1He-KExbsz5x7wc5iPL45rgw%40mail.gmail.com.


Re: For Brent ...

2024-09-10 Thread John Clark
On Mon, Sep 9, 2024 at 1:19 PM Alan Grayson  wrote:


> *> If you're interested in the Tao instead of laughing at it, google
> "chakra'. AG*


*If you're really interested in The Tao then forget chakra and read "The
Tao Is Silent" by Raymond Smullyan. It's one of the most brilliant books
I've ever read. *

The Tao Is Silent


John K ClarkSee what's on my new list at  Extropolis

rsb

-- 
You received this message because you are subscribed to the Google Groups 
"Everything List" group.
To unsubscribe from this group and stop receiving emails from it, send an email 
to everything-list+unsubscr...@googlegroups.com.
To view this discussion on the web visit 
https://groups.google.com/d/msgid/everything-list/CAJPayv02U0aeii028a4Qv3a7a786T90O8AcvA6hFF%3DkXCJMVwA%40mail.gmail.com.


Re: Amoeba's Secret openly available under CC-BY license

2024-09-10 Thread John Clark
On Mon, Sep 9, 2024 at 2:14 PM Brent Meeker  wrote:

*> I generally agree with John, but I would point out that computation is a
> physical process that realizes a mathematical process. *
>

*I think it would be more accurate to say it realizes an informational
process because mathematics is just a small subset of logic. In Boolean
arithmetic  1+1 =1. And the very word "process" implies a procedure that
causes a change with the passage of time, but without physics nothing can
change. No matter how good a mathematics book is it will never change, and
if it's just sitting on the shelf and nothing physical, human or AI, ever
reads it then it will not cause anything else to change either.*


> *> Sure it's more complicated because it depends on the physics, but that
> is incidental to the computation.*


*The particular physics used in a computation is incidental BUT the use
of SOME variety of physics (mechanical gears rods and pulleys, biological
nerves, vacuum tubes, transistors, integrated circuits, quantum computers
etc) is NOT incidental because information is physical, so if you want to
process it mathematics is not enough, you need physics. If that were not
true Nvidia then would go broke as would the entire semiconductor industry.*

 John K ClarkSee what's on my new list at  Extropolis

nsi

-- 
You received this message because you are subscribed to the Google Groups 
"Everything List" group.
To unsubscribe from this group and stop receiving emails from it, send an email 
to everything-list+unsubscr...@googlegroups.com.
To view this discussion on the web visit 
https://groups.google.com/d/msgid/everything-list/CAJPayv3uZLOvE1%2B%3DkbkrCGibPdtLipoYzd2KxzEtwL1P%3DNor5Q%40mail.gmail.com.


Re: Length contraction in Special Relativity

2024-09-10 Thread John Clark
On Mon, Sep 9, 2024 at 9:54 PM Alan Grayson  wrote:

*> the rate of expansion of the universe appears to be increasing, *


*Yes but even if the rate of expansion stopped increasing it would still be
true that some galaxies we can see today we won't be able to see tomorrow. *

*> is it increasing fast enough to cause galaxies in our local group to
> become part of the UNobservable universe?*


*We are already part of the unobservable universe to galaxies that we
cannot see. None of the galaxies in the local group are moving away from us
because the local group is gravitationally bound together, that is to say
the mutual gravitational attraction between its members is strong enough to
overcome the general expansion of the universe. But there are only about 80
galaxies in our local group, the three largest in order of size are
Andromeda, the Milky Way and the Triangulum Galaxy. 77 are just dwarf
galaxies.  Except for those, all the other galaxies in the universe are
moving away from us.*


>
> * > I conjecture that the UNobservable universe came into being with
> Inflation. If so, it must have been initially finite in spatial extent*


*If the entire universe, observable plus unobservable, is infinite today
then it must've been infinite even before inflation started, it must've
been infinite from the first Planck Time Instant of its creation. I think
most cosmologists would say that at the largest level space is curved into
some unknown shape but inflation has flattened it out so much the curvature
is too small to ever be detected.  *

*They used to say there was a strict relationship between the amount of
mass/energy in the universe and its shape, if it was less than a certain
figure it was positively curved like a sphere, if it was over that figured
it was negatively curved like a saddle, and if it was EXACTLY at that
figure it was flat; but after the discovery of Dark Energy things became a
lot more complicated and they knew that simple relationship could not be
true.   *

*By the way, there is some indication that Dark Energy is getting weaker
and thus the acceleration of the universe is slowing down, but there's not
enough evidence to claim a discovery and get a Nobel prize; it only has 3.9
sigma, enough to be very exciting but you need at least 5 sigma to claim a
discovery. If that turns out to be true then all bets are off and we have
no idea what the distant future will be like, for all we know after Dark
Energy drops to zero it may turn negative and Dark Energy might start
slowing down the expansion of the universe. Nobody knows.*

*Dark Energy May Be Weakening, Major Astrophysics Study Finds*


  John K ClarkSee what's on my new list at  Extropolis

ba0

-- 
You received this message because you are subscribed to the Google Groups 
"Everything List" group.
To unsubscribe from this group and stop receiving emails from it, send an email 
to everything-list+unsubscr...@googlegroups.com.
To view this discussion on the web visit 
https://groups.google.com/d/msgid/everything-list/CAJPayv3k8%3DFzCuYM012RpzAbsCXDAXnhTWyfsmWT0uJprjBD7Q%40mail.gmail.com.


Re: Length contraction in Special Relativity

2024-09-09 Thread John Clark
On Sun, Sep 8, 2024 at 9:13 PM Alan Grayson  wrote:

*>> if you extended a line from you to any point on the sky it would
>> eventually hit the center of a star, and so every point on the nighttime
>> sky would be as bright as the sun. But that's not what we observe.*
>
>
> *> As for the unobservable part of the universe, moving away at faster
> than light speed, I conjecture that Inflation is the cause.*
>

*I'm not talking about inflation, just the normal everyday expansion of the
universe, which has been known since the mid-1920s, means that stars  a
finite distance away are moving away from us faster than the speed of light
, and so the light from them will never reach us.  *

*> if we run the clock backward, they would eventually come back into
> view, *
>

*Yes there are stars that we can see today that we won't be able to see
tomorrow, 1 trillion years from now we won't be able to see any stars
except those that are in the Milky Way because those stars are
gravitationally bound together.  *

*> I disagree with your final conclusion. Even if the universe is infinite,
> many stars that are directly in our line of sight, might be too faint to be
> seen, as is the case of nearby brown dwarf stars, which comprise 50% of
> stars in our relatively nearby neighborhood, but too faint to see.*
>

*Sirius A is the brightest star in the sky but it has a companion, Sirius
B, which is hotter and, because the light emitted of a hot object  is
proportional to the size of the object and to the fourth power of the
temperature, is much much brighter, and yet it is impossible to see unless
you have a fairly large telescope. That is because although its light is
very intense Sirius B is far smaller than Serious A. One has a diameter of
about 1,000,000 miles while the other has a diameter of only 6800 miles, so
even though it's very intense the total amount of light given off is much
less than Sirius A. *

  John K ClarkSee what's on my new list at  Extropolis

abc

-- 
You received this message because you are subscribed to the Google Groups 
"Everything List" group.
To unsubscribe from this group and stop receiving emails from it, send an email 
to everything-list+unsubscr...@googlegroups.com.
To view this discussion on the web visit 
https://groups.google.com/d/msgid/everything-list/CAJPayv0GP%2Br7r5VwW3tjXrcdxNwL92nt6LsWRk%2BgWax1ucHm%3Dg%40mail.gmail.com.


Re: Amoeba's Secret openly available under CC-BY license

2024-09-09 Thread John Clark
On Sun, Sep 8, 2024 at 7:10 PM Russell Standish 
wrote:


> *>  Most of the complexity of a billiard ball is accidental anyway.*


*Even if you ignore the internal chemistry of a billiard ball a
mathematical sphere is only an approximation of a billiard ball. It would
take very little information to perfectly describe the shape of a sphere,
but it would take vastly more information to perfectly describe the shape
of a physical billiard ball. And as I said before, simulated physical
objects are simpler than real physical objects. Mathematics is the language
of reality but I've never heard of a description of an object being more
complex than the object itself. *

>
* > The idea is that if consciousness is a computational thing *


*I Think it's a brute fact that consciousness is the way data feels when it
is being processed intelligently, and the argument I gave in the above does
not apply to consciousness because it is not a physical object, it is not
even a noun, it is an adjective. Intelligence is the way atoms behave when
they are arranged in ways that can produce the same results that a Turing
Machine can. And consciousness is the way data feels when it is being
processed intelligently.*

*> (ie) mathematical),*


*No. Mathematics can describe computation, but it is not computation.
That’s why the semiconductor industry exists, software alone is not
sufficient, in fact, software alone can’t do anything.  If you actually
want to DO something, if you want something to change over an interval of
time, then matter is required. That's why the information in a book can't
do anything if it's just sitting on a shelf, that information can only
cause something to change if a person or, as we've seen very recently, an
AI, reads it.  And both the person and the AI are made of atoms. And atoms
are physical.  *

*Computation involves the manipulation of information, and the minimum
amount of energy needed to perform a calculation is greater than zero.
Also, the amount of information that you can stuff into a volume of space
is finite, if there is too much information then the volume turns into a
Black Hole where the information, if it still even exists, is
inaccessible. So information is physical and computation is a physical
process. *

*> **then phenomena (ie physics) is entirely due to random splitting*


*Schrodinger's wave function is NOT random, it is 100% deterministic. And
if you wanted to sum up Hugh Everett's Many Worlds theory in the fewest
words possible it would simply be "everything follows Schrodinger's wave
function, even the entire universe". So to a mind vast enough to comprehend
the entire Universal Wave Function nothing would be random. *


> * > So either physics is arithemetic, with true randomness built in, or*
>

*Or arithmetic is a subset of physics.  *


> *> consciousness is not computational.*
>

*I keep asking this question but I never receive an answer: "if
consciousness is not computational then how did Darwin's theory of natural
selection manage to produce at least one conscious being, me, ., and
possibly you too?"  *

 John K ClarkSee what's on my new list at  Extropolis

pu2

-- 
You received this message because you are subscribed to the Google Groups 
"Everything List" group.
To unsubscribe from this group and stop receiving emails from it, send an email 
to everything-list+unsubscr...@googlegroups.com.
To view this discussion on the web visit 
https://groups.google.com/d/msgid/everything-list/CAJPayv3T_taFNH%2BtBXirsyco9Uaty7%2BgKLqu%3DLQSggqK%2B8Oeqw%40mail.gmail.com.


Re: Length contraction in Special Relativity

2024-09-08 Thread John Clark
On Sun, Sep 8, 2024 at 1:23 AM Brent Meeker  wrote:

*> Given that already since Olaf Römer's observations of 1676 it has been
> known that light propagates at a finite speed, it would have been possible
> more than 300 years ago to conclude that objects moving at nearly the speed
> of light must look distorted. Surprisingly, no such conclusions have been
> drawn in the framework of classical physics. *


*True. They could also have concluded in 1676 that the universe must be a
finite number of miles across, or created a finite number of years ago, or
space itself must be expanding and so very distant stars must be moving
away from us faster than the speed of light so the light from them will
never reach us. I say that because if none of those three things were true
then if you extended a line from you to any point on the sky it would
eventually hit the center of a star, and so every point on the nighttime
sky would be as bright as the sun. But that's not what we observe.*


*And as early as 1687 (maybe even earlier) when Isaac Newton published
Principia Mathematica, all the mathematics needed to develop Special
Relativity was there, all that was needed was to make the assumption that
nothing can go faster than light, and that it's measured speed was always
the same for all observers. I don't think Newton would've minded the idea
that the universe contains a speed limit, he was after all very
uncomfortable with the idea of action at a distance, but it would've
probably taken a lot of convincing for him to believe that all observers
would see light moving at exactly the same speed. *

*If I was talking to Newton I'd point out how odd it was that the concept
of mass can be defined in two apparently unrelated ways, the amount of
gravitational force the mass produces, and how difficult it is to change
the velocity of that mass. And I'd ask him if he thought it was just a
coincidence that those two things gave the same value and was the reason
that heavy things and light things fell with the same speed in a
gravitational field . And I'd mentioned the thought experiment about a man
and a falling elevator and a man in an accelerating rocket. *

*As for General Relativity, I don't think anybody could've come up with
that more than two or three decades before Einstein did, before that the
mathematics just wasn't sufficient. *

  John K ClarkSee what's on my new list at  Extropolis

jws








>

-- 
You received this message because you are subscribed to the Google Groups 
"Everything List" group.
To unsubscribe from this group and stop receiving emails from it, send an email 
to everything-list+unsubscr...@googlegroups.com.
To view this discussion on the web visit 
https://groups.google.com/d/msgid/everything-list/CAJPayv1tfpRvonFj5m_KV0hvdJR9M%2BHi6F2vxT24miSYLAPw_w%40mail.gmail.com.


Re: Amoeba's Secret openly available under CC-BY license

2024-09-08 Thread John Clark
On Sat, Sep 7, 2024 at 11:16 PM Alan Grayson  wrote:

*> I've been reading Marchal's de facto autobiography. It's hugely
> impressive, leading me to be more open to his main conclusion, IIUC, that
> arithmetic is at the core of reality, yes, physical reality. AG*
>

*If a mathematical sphere is the fundamental reality and a physical
billiard ball is just an approximation of one, then why is a billiard ball
astronomically more complex than the sphere? Wouldn't you expect an
approximation (or a simulation) to be simpler than the real thing? I think
it would be more accurate to say that a meteorologist's mathematical model
is an approximation of a physical hurricane, and a physical hurricane is
NOT an approximation of a meteorologist's mathematical model. *

*Mathematics is the language of physics but mathematics is not physics.
That's why Physics is more fundamental than mathematics, and that's why you
can't get milk out of the English language word "COW".*

John K ClarkSee what's on my new list at  Extropolis

swo

-- 
You received this message because you are subscribed to the Google Groups 
"Everything List" group.
To unsubscribe from this group and stop receiving emails from it, send an email 
to everything-list+unsubscr...@googlegroups.com.
To view this discussion on the web visit 
https://groups.google.com/d/msgid/everything-list/CAJPayv3-uxxKs21iFHw7HN4Mm95T65k7KS_DSHAtzM8dW3kXtw%40mail.gmail.com.


Nick Bostrom on AI’s Radical Potential

2024-09-05 Thread John Clark
>From the article:

*"If you think of all those possible, physically possible technologies that
maybe human civilization would invent if we had like 20,000 years to work
on it with human scientists, maybe we would have space colonies and perfect
virtual reality and cryonics patients could be thawed up and cures for
cancer, all these kinds of science fictiony technologies. But that don’t
break any laws of physics. I think all of that might become available
within a year or a few years after we have superintelligence.”*

*Telescoping Tomorrow: Nick Bostrom on AI’s Radical Potential*


John K ClarkSee what's on my new list at  Extropolis

fta

-- 
You received this message because you are subscribed to the Google Groups 
"Everything List" group.
To unsubscribe from this group and stop receiving emails from it, send an email 
to everything-list+unsubscr...@googlegroups.com.
To view this discussion on the web visit 
https://groups.google.com/d/msgid/everything-list/CAJPayv2A%3D-7x%3DYkUhjCtPf0uCcEpuNSg9TxKa0bBtOz51%3D76Tg%40mail.gmail.com.


The first prototype Nuclear Clock

2024-09-05 Thread John Clark
*Concerning Yesterday's issue of the journal Nature: *


*“With this first prototype, we have proven: Thorium can be used as a
timekeeper for ultra-high-precision measurements,” says Thorsten Schumm, an
author of the study. “All that is left to do is technical development work,
with no more major obstacles to be expected.” The researchers predict that
the nuclear clock should surpass the precision of atomic clocks in two to
three years, and should also be more portable and stable. If they take over
the role, we could be in for much faster and more reliable communications,
internet, GPS, and other technologies. They could even help scientists
probe the fundamentals of physics and aid things like the search for Dark
Matter, determining if the fundamental "constants" of nature are really
constant, and in the detection of gravitational waves.*

*Progress on nuclear clocks*


John K ClarkSee what's on my new list at  Extropolis

fcn

-- 
You received this message because you are subscribed to the Google Groups 
"Everything List" group.
To unsubscribe from this group and stop receiving emails from it, send an email 
to everything-list+unsubscr...@googlegroups.com.
To view this discussion on the web visit 
https://groups.google.com/d/msgid/everything-list/CAJPayv0X8vHK2RSR-S_PimU1eC%3DKLux%2B9SGNm5%3Dw41pbN61Vhg%40mail.gmail.com.


Data Centers in Orbit?

2024-09-05 Thread John Clark
Will Data Centers in Orbit Launch a New Phase of Sustainability?


John K ClarkSee what's on my new list at  Extropolis

ein

-- 
You received this message because you are subscribed to the Google Groups 
"Everything List" group.
To unsubscribe from this group and stop receiving emails from it, send an email 
to everything-list+unsubscr...@googlegroups.com.
To view this discussion on the web visit 
https://groups.google.com/d/msgid/everything-list/CAJPayv3Rykh%3DdGp3zBfBMedxA_tmPtUD3reRAb6WnO5g1qNRQw%40mail.gmail.com.


Re: For Brent ...

2024-09-04 Thread John Clark
On Wed, Sep 4, 2024 at 8:08 AM Alan Grayson  wrote:


> *> it seems that the metric tensor FIELD is NOT well defined. AG*



*The metric tensor encodes spacetime curvature, and for every point in
spacetime that you can name I can give you a 4x4 matrix of unique
computable numbers that defines the curvature at that point. What's
ambiguous about that? It's true that I can't do that for points in
spacetime that you cannot name, but that is not a problem because you
cannot get experimental results from points that you cannot name. *

John K ClarkSee what's on my new list at  Extropolis
<https://groups.google.com/g/extropolis>
wai





>
> On Wednesday, September 4, 2024 at 6:02:17 AM UTC-6 John Clark wrote:
>
>> On Tue, Sep 3, 2024 at 6:44 PM Alan Grayson  wrote:
>>
>> * > I fail to see how your comments relate to the possibly ambiguous
>>> concept of the latter. The metric tensor field seems ambiguously defined.*
>>
>>
>> *A N dimensional space is composed of an uncountable number of real
>> numbers but it can be unambiguously defined by just N countable rational
>> numbers, you can pair them up one to one. This is possible because there is
>> only a countably infinite number of COMPUTABLE real numbers, the same rank
>> of infinity as the rational numbers. So you can in effect give a rational
>> number name to every real number you are able to find on the number line.
>> You can do this even for a number such as π which is not only irrational,
>> it's transcendental, because it is also computable. You can use an infinite
>> series to get arbitrarily close to π.  *
>>
>> *The vast majority of numbers on the number line are NOT computable (and
>> have no name) but that's not really a problem despite the fact that the
>> vast majority of numbers on the number line are NOT computable because,
>> except for Chaitin's Omega Number, every number that a mathematician has
>> ever heard of is a computable number. Computable numbers can have names,
>> uncomputable numbers can not.*
>>
>

-- 
You received this message because you are subscribed to the Google Groups 
"Everything List" group.
To unsubscribe from this group and stop receiving emails from it, send an email 
to everything-list+unsubscr...@googlegroups.com.
To view this discussion on the web visit 
https://groups.google.com/d/msgid/everything-list/CAJPayv3MbDFig6j5-e-J2ZKACFe5pCSnefA3nrCeXqb_SHHJbA%40mail.gmail.com.


Re: For Brent ...

2024-09-04 Thread John Clark
On Tue, Sep 3, 2024 at 6:44 PM Alan Grayson  wrote:

* > I fail to see how your comments relate to the possibly ambiguous
> concept of the latter. The metric tensor field seems ambiguously defined.*


*A N dimensional space is composed of an uncountable number of real numbers
but it can be unambiguously defined by just N countable rational numbers,
you can pair them up one to one. This is possible because there is only a
countably infinite number of COMPUTABLE real numbers, the same rank of
infinity as the rational numbers. So you can in effect give a rational
number name to every real number you are able to find on the number line.
You can do this even for a number such as π which is not only irrational,
it's transcendental, because it is also computable. You can use an infinite
series to get arbitrarily close to π.  *

*The vast majority of numbers on the number line are NOT computable (and
have no name) but that's not really a problem despite the fact that the
vast majority of numbers on the number line are NOT computable because,
except for Chaitin's Omega Number, every number that a mathematician has
ever heard of is a computable number. Computable numbers can have names,
uncomputable numbers can not.*
  John K ClarkSee what's on my new list at  Extropolis


und

-- 
You received this message because you are subscribed to the Google Groups 
"Everything List" group.
To unsubscribe from this group and stop receiving emails from it, send an email 
to everything-list+unsubscr...@googlegroups.com.
To view this discussion on the web visit 
https://groups.google.com/d/msgid/everything-list/CAJPayv13yFrwMMLBu7og_E%2BDVv9ab%3DTTJaB2wY%2B4kekuFJ6VuQ%40mail.gmail.com.


Re: For Brent ...

2024-09-03 Thread John Clark
On Tue, Sep 3, 2024 at 3:58 PM Alan Grayson  wrote:

*> I don't think you understand the issue. Velocities greater than c in the
> underlying spacetime manifold are allowed in the construction of the
> tangent plane,*


I don't think you understand that the things that mathematics allows and
the things that the laws of physics allow are not necessarily the same
thing. Mathematics allows for the existence of Newtonian physics, and
Newton allows objects to travel much faster than the speed of light, but
Einstein taught us that the laws of physics forbid it.

Nearly all new theories in physics are mathematically consistent, but
nearly all new theories in physics are also eventually proven to be wrong.

 John K ClarkSee what's on my new list at  Extropolis

wpb

-- 
You received this message because you are subscribed to the Google Groups 
"Everything List" group.
To unsubscribe from this group and stop receiving emails from it, send an email 
to everything-list+unsubscr...@googlegroups.com.
To view this discussion on the web visit 
https://groups.google.com/d/msgid/everything-list/CAJPayv03Kui2TLEO7KPPSh3rZSMTYtR1xva8W8TZa-Vtz7HFsg%40mail.gmail.com.


Re: For Brent ...

2024-09-03 Thread John Clark
On Tue, Sep 3, 2024 at 10:07 AM Alan Grayson  wrote:

*> While spacetime might not have an infinite set of events, countable or
> uncountable, the tangent space is constructed via a vector space with at
> least a countable number of elements. *


Even though there are an uncountably infinite number of real numbers but
only a countable number of rational numbers, you can always find a rational
number that is arbitrarily close to any real number, provided that the real
number in question is computable. Thus the thing that you're so worried
about is not a concern for a physicist because, at least so far as anybody
knows, there is no physical experiment you can perform that can reveal the
difference between the countable infinite and the uncountable infinite.

Mathematics is the language of physics but mathematics is not physics. And
any language can be used for both fiction and nonfiction, so it could be
that the hierarchies of infinity and even the very concept of infinity is
the mathematical equivalent of a Harry Potter novel. Mathematical
consistency may not be enough to ensure physical reality.

 John K ClarkSee what's on my new list at  Extropolis

acr




>

-- 
You received this message because you are subscribed to the Google Groups 
"Everything List" group.
To unsubscribe from this group and stop receiving emails from it, send an email 
to everything-list+unsubscr...@googlegroups.com.
To view this discussion on the web visit 
https://groups.google.com/d/msgid/everything-list/CAJPayv0qcsG4eYQiT_QyFRbipkHJTxncOBqXWHGhNXso%3D%2B11EQ%40mail.gmail.com.


Re: For Brent ...

2024-09-03 Thread John Clark
On Sat, Aug 31, 2024 at 10:48 PM Alan Grayson 
wrote:

* > please explain how the metric tensor can be defined unambiguously at
> some point P on the underlying manifold, spacetime, if there is an
> uncountable set of pairs on a vector space on the tangent space at some
> point P on which the metric tensor is defined*



If, as I suspect, your interest is physics and not pure mathematics then
it's a non-issue. The fact is nobody is even sure that 4D space-time
contains an infinite number of points, for all we know it may only contain
an astronomical number to an astronomical power number of points. That's
undoubtedly a very big number but it's no closer to being infinite than the
number one is.

And even if 4D space-time does contain an uncountabley infinite number of
points, if you simplify your physical theory by assuming there is only a
countably infinite number of points it will have a negligible effect on
your theory; that is to say you could make the discrepancy between what
your theory predicts will happen and what you actually observed to happen
in experiments to be arbitrarily small. I am not aware of any physical
theory in which the difference between countable infinity and uncountable
infinity leads to different experimentally testable predictions.

 John K ClarkSee what's on my new list at  Extropolis

n4x

-- 
You received this message because you are subscribed to the Google Groups 
"Everything List" group.
To unsubscribe from this group and stop receiving emails from it, send an email 
to everything-list+unsubscr...@googlegroups.com.
To view this discussion on the web visit 
https://groups.google.com/d/msgid/everything-list/CAJPayv2R0Zp%3DAhZZ93_nk%3DcCYab8_T1sh9s-LuZpX67c0boRag%40mail.gmail.com.


NYTimes.com: America Must Free Itself from the Tyranny of the Penny

2024-09-01 Thread John Clark
Explore this gift article from The New York Times. You can read it for free
without a subscription.

America Must Free Itself from the Tyranny of the Penny

Few things symbolize our national dysfunction as much as this accursed
coin, which we mint by the millions because it’s too worthless to spend.

https://www.nytimes.com/2024/09/01/magazine/worthless-pennies-united-states-economy.html?unlocked_article_code=1.HU4.0u1V.6UJ3jF2JUbH6&smid=em-share

-- 
You received this message because you are subscribed to the Google Groups 
"Everything List" group.
To unsubscribe from this group and stop receiving emails from it, send an email 
to everything-list+unsubscr...@googlegroups.com.
To view this discussion on the web visit 
https://groups.google.com/d/msgid/everything-list/CAJPayv3%2BEHWoHEJoQQDVDJhEuPVWJgxZQe3N%2BCmo_szjiQ3NXQ%40mail.gmail.com.


Crypto is dominating corporate election spending

2024-08-30 Thread John Clark
*Crypto is dominating corporate election spending* 


John K ClarkSee what's on my new list at  Extropolis

32h

-- 
You received this message because you are subscribed to the Google Groups 
"Everything List" group.
To unsubscribe from this group and stop receiving emails from it, send an email 
to everything-list+unsubscr...@googlegroups.com.
To view this discussion on the web visit 
https://groups.google.com/d/msgid/everything-list/CAJPayv2331zFDAqBQ7ONTsW6-0vHcvb8gJQmfR3aHWo2nJuUiA%40mail.gmail.com.


Re: I am God

2024-08-30 Thread John Clark
On Fri, Aug 30, 2024 at 3:48 AM 'Cosmin Visan' via Everything List <
everything-list@googlegroups.com> wrote:

> Why is this knowledge not taught in schools ?
>

Because most people are not crackpots, not even in Florida despite the
governor's incessant efforts to increase the number of ignoramuses in his
state.

Major Publishers, Authors Guild Sue Over New Florida Book Banning Law


 John K ClarkSee what's on my new list at  Extropolis

rds







>

-- 
You received this message because you are subscribed to the Google Groups 
"Everything List" group.
To unsubscribe from this group and stop receiving emails from it, send an email 
to everything-list+unsubscr...@googlegroups.com.
To view this discussion on the web visit 
https://groups.google.com/d/msgid/everything-list/CAJPayv0NviR9UZpJYbN%2Buh%3DNMcEM8ifrRzrb6PdNZk0M0%2BiB%2Bg%40mail.gmail.com.


Re: Realization that the physical world doesn't exist

2024-08-30 Thread John Clark
On Fri, Aug 30, 2024 at 3:47 AM 'Cosmin Visan' via Everything List <
everything-list@googlegroups.com> wrote:

*>Why do people not realize that the physical world doesn't exist,*


OK. But how would things be different if the physical world did exist?

*>that is just an idea in consciousness ?*
>

Maybe because we are not conscious and you are the only conscious being in
the universe.  Maybe that's why we think you are a crackpot.

John K ClarkSee what's on my new list at  Extropolis

vcd

-- 
You received this message because you are subscribed to the Google Groups 
"Everything List" group.
To unsubscribe from this group and stop receiving emails from it, send an email 
to everything-list+unsubscr...@googlegroups.com.
To view this discussion on the web visit 
https://groups.google.com/d/msgid/everything-list/CAJPayv20Qyn5HjJF2Fh_N8VgDryEzCGvjCANBzYJt300c-jXAA%40mail.gmail.com.


NYTimes.com: Trump Sees a World on Fire, and Says He Knows Who’s to Blame

2024-08-29 Thread John Clark
Explore this gift article from The New York Times. You can read it for free
without a subscription.

Trump Sees a World on Fire, and Says He Knows Who’s to Blame

Former President Donald J. Trump blames President Biden for crises around
the globe. But the reality is that presidents inherit a world already
hurtling through history.

https://www.nytimes.com/2024/08/28/us/politics/trump-biden-iran-gaza-afghanistan-ukraine.html?unlocked_article_code=1.Gk4.Qysa.uykam3wPtLKS&smid=em-share

-- 
You received this message because you are subscribed to the Google Groups 
"Everything List" group.
To unsubscribe from this group and stop receiving emails from it, send an email 
to everything-list+unsubscr...@googlegroups.com.
To view this discussion on the web visit 
https://groups.google.com/d/msgid/everything-list/CAJPayv26WGq8jmHTnKm0wkt1VbiJxit82-7W_PX%2BHhbsj2XMYA%40mail.gmail.com.


NYTimes.com: Telegram Becomes Free Speech Flashpoint After Founder’s Arrest

2024-08-26 Thread John Clark
Explore this gift article from The New York Times. You can read it for free
without a subscription.

Telegram Becomes Free Speech Flashpoint After Founder’s Arrest

Pavel Durov, the founder of the app, which has more than 900 million users,
was taken into custody by the French authorities.

https://www.nytimes.com/2024/08/25/technology/pavel-durov-telegram-detained-france.html?unlocked_article_code=1.F04.sHbL.6qc0UQHqAybU&smid=em-share

-- 
You received this message because you are subscribed to the Google Groups 
"Everything List" group.
To unsubscribe from this group and stop receiving emails from it, send an email 
to everything-list+unsubscr...@googlegroups.com.
To view this discussion on the web visit 
https://groups.google.com/d/msgid/everything-list/CAJPayv2U%2B5wgxtPG2%3DQ92N7zZivbrEUPbkP%2BKcBMVHSP6UMkKg%40mail.gmail.com.


Re: Politics, AI and Quantum Computing

2024-08-25 Thread John Clark
On Sat, Aug 24, 2024 at 3:14 PM PGC  wrote:

*> Regarding politics, some reflections on the Democratic Convention from
> my perspective: Watching the proceedings I had time to watch, I couldn't
> help but notice a focus on performance over substance.*
>
*Hillary Clinton lost in 2016 because she put more emphasis on substance
than performance, she thought she could win with position papers and
PowerPoint slides. She was wrong. When Donald Trump speaks there is often
no substance at all and he just rambles on about sharks getting
electrocuted and low flush toilets and lightbulbs and crowd size and "the
late great Hannibal Lecter", but Trump always puts on a good show; or at
least that's what everybody tells me, they tell me the man has charisma.
Personally I don't see it but there's no disputing matters of taste. Some
people find cockroaches to be cute. *

*On occasion Trump does actually say something of substance but when he
does you can be certain of one thing, that substance will be UGLY. *

 >*Is it wise to rally behind a candidate, post-Biden, without any public
> debate or transparent selection process—especially when this candidate was,
> until recently, widely seen as a disappointment?*


*Yes I think it's very wise to rally around Harris because it's not
important if she turns out to be a disappointing or mediocre president,
it's not even important if she turns out to be a Nixon level bad president
because we can survive that. The important thing is that she is unlikely to
become an apocalyptically catastrophic president like Donald Trump would be
if he gets a second term. His first term was bad enough! It's far more
important to avoid a very bad president than it is to elect a very good
president, that's because there's a limit to the amount of good a president
can do even if she's a transcendental genius and a saint, but there is no
bottom to bad. *

*> Remember how late Obama’s endorsement came?*
>

*I don't think that was because of any lack of confidence in Harris, but
because he thought she should win the nomination by fighting for it. I
agree with Obama about that, but the party chose another way and I'll just
have to live with it.*

*> At least his speech included a touch of sobriety. If I recall correctly,
> he cautioned against confusing the euphoria in the Chicago arena with the
> sentiment of the entire nation.*
>

*I agree with that too. It's encouraging that Harris is doing much better
in the polls than Biden but it's important to remember that Trump has
historically always done much better than what the polls had predicted he
would do. I think that's because many people are  embarrassed to admit to a
pollster that they're going to vote for Trump. I don't blame them, if I was
going to vote for Trump I'd be embarrassed to admit it too.  *
  John K ClarkSee what's on my new list at  Extropolis

ite




>

-- 
You received this message because you are subscribed to the Google Groups 
"Everything List" group.
To unsubscribe from this group and stop receiving emails from it, send an email 
to everything-list+unsubscr...@googlegroups.com.
To view this discussion on the web visit 
https://groups.google.com/d/msgid/everything-list/CAJPayv2bF2ThcLubiQJ_8XgjJvDaA1BR3pUC12tdWtGN97ZtBg%40mail.gmail.com.


Can AI Scaling Continue Through 2030?

2024-08-24 Thread John Clark
*There's a very interesting paper at: *


*Can AI Scaling Continue Through 2030?
*


*They highlight 4 things that might slow down the growth of AI but, spoiler
alert, despite these constraints the answer they come to at the end is
"yes".  They conclude that by 2030 AI training runs of 10^29 flops will be
happening, to put it in perspective, that would be 10,000 times as large as
GPT4's most advanced model. This despite 4 things that might slow things
down, they are Power Constraints, Chip Manufacturing Capacity, Data
Scarcity and the Latency Wall.*

  Power constraints

*The FLOP/s per watt efficiency of GPUs used for AI training has been
increased by 1.28 times each year between 2010 and 2024, if continued, and
they see no reason to believe it won't,  training runs will be 4 times more
energyefficient by the end of the decade. Also there is near universal
agreement that in the future the neural net training of AIs will switch
from 16 bit precision to 8 bit, and that alone would double the
efficiency.  They conclude that in 2030 it would take about 6 gigawatts for
a year to teach an AI that was 10,000 times the size of GPT 4, that may
seem like a lot but the total power capacity of the US is about 1,200
gigawatts. *

  Chip manufacturing capacity

*There is considerable uncertainty about this, the best estimate they could
come up with is that between 20 million to 400 million Nvidia H100
equivalent GPUs will be manufactured in 2030, and that would be sufficient
to allow for training runs between 5000 and 250,000 times larger than
GPT4's training run.*

 Data scarcity

*The largest training data set  to have been used in training is 15
trillion tokens of publicly available text. The entire World Wide Web
contains about 500 trillion tokens, and the nonprofit "CommonCrawl" alone
has about 100 trillion tokens. If you include private data that figure
could go as high as 3000 trillion tokens. Also, synthetic data is proven to
be increasingly useful, especially in fields like mathematics, games and
software engineering, because they are all in affect asking NP
questions; that is to say questions that may be very difficult to find the
answers too but are very easy to determine if the proposed answers are
indeed correct. *

* Latency wall*

*The authors believe this will have little effect before 2030, but may need
to be considered after that when we reach the 10^31 flop level. The idea is
that during learning as neural networks get larger the time required to
pass forward and backward through the system increases. This problem can be
ameliorated by finding all the things that can be done in parallel into
something they call "pods", but you can reach a point of diminishing
returns if the size of the pods gets too large then things must be
processed sequentially. *

*There are ways to get over this wall but it will require changes to the
basic topology of the neural nets.   *

*Instead of relying on frequent communication between different parts of
the model, computations can be organized so that more work is done locally
within each computational unit. *

*Asynchronous communication can be used, this is where nodes can continue
processing without waiting for data from other parts of the network. *

*Specialized hardware such as Tensor Processing Units that have low latency
and high bandwidth can be used. *

*Designing the network  to reduce the number of hops data needs to take can
also help mitigate latency. *

*Data compression can be used to reduce the amount of information needed to
be transferred within the system. *
*There are even advanced algorithms that can work with old "stale"
information without significant loss in performance. *

*The bottom line is  the authors predict that in the next few years
hundreds of billions or trillions of dollars will be spent on AI, and it
will become  "the largest technological project in the history of
humankind".*

*  John K Clark*

-- 
You received this message because you are subscribed to the Google Groups 
"Everything List" group.
To unsubscribe from this group and stop receiving emails from it, send an email 
to everything-list+unsubscr...@googlegroups.com.
To view this discussion on the web visit 
https://groups.google.com/d/msgid/everything-list/CAJPayv2f4QQr95FOGaDRqiLhWc-5PaSWvSksuCTADNodXHu6Fg%40mail.gmail.com.


Re: Dyson Spheres and the Fermi Paradox

2024-08-20 Thread John Clark
On Tue, Aug 20, 2024 at 1:36 AM Brent Meeker  wrote:

> >> Just ONE bacteria placed in a sterile body of water the size of the
>> Pacific Ocean could spread so that there were millions of bacteria in every
>> square inch of it in just a few weeks, provided the ocean had the
>> correct nutrients. And a Von Neumann probe would be far far less picky
>> about what it considers the "correct nutrients" than a bacteria.
>
> *> Bacteria are killed by radiation and a von Neumann probe will be
> too...eventually. *
>

With redundancy and a rapid self repair mechanism a Von Neumann Probe can
have an arbitrarily low failure rate. That's how the bacteria Deinococcus
radiodurans is able to live inside a nuclear reactor; depending on
conditions each bacteria has between 4 and 10 copies of its genome and can
repair any damage to it very rapidly. It can live a happy life despite
being exposed to 3000 times as much radiation that would kill a human being
in just a few hours.


*> I think you are far too sanguine about how easy it will be to build such
> a thing and to know it will work for the requisite time to build replicas. *
>

I disagree with you about that, and so would John Von Neunmann. And so
would Enrico Fermi, in fact that's why he originally posed the question "*where
is everybody?*". Only one resolution to the Fermi Paradox (a.k.a. The Great
Silence) makes any sense to me.  ET doesn't exist.

*> There's a scifi story about a part of the galaxy in which vNp have been
> built and evolved and are now fighting one another for resources.*
>

If it came to the point where Von Neumann Probes had to fight each other
for the most useful elements, like hydrogen oxygen carbon and nitrogen, the
first third fourth and sixth most common elements in the universe, then
that epic battle would be easily observable by us even if it occurred a
long time ago in a galaxy far far away.  But we see no sign of such a
thing.

 John K ClarkSee what's on my new list at  Extropolis

sav


>

-- 
You received this message because you are subscribed to the Google Groups 
"Everything List" group.
To unsubscribe from this group and stop receiving emails from it, send an email 
to everything-list+unsubscr...@googlegroups.com.
To view this discussion on the web visit 
https://groups.google.com/d/msgid/everything-list/CAJPayv1fq1v3XAwZmhxeSdFm%2B7TX3ePK4RNuJ2%3DgdO%2BLb_GsJw%40mail.gmail.com.


Re: Dyson Spheres and the Fermi Paradox

2024-08-19 Thread John Clark
On Mon, Aug 19, 2024 at 8:06 PM Brent Meeker  wrote:

*>>> **if the Milky way has on produced a handful of vNps then it's not so
>> implausible that none of have succeeded or even tried.*
>>
>
> >> I don't know how you figure that. It seems ridiculously implausible to
> me because it only takes one.
>
> *> But how many trying does it take to get one succeeding.*
>

Just ONE bacteria placed in a sterile body of water the size of the Pacific
Ocean could spread so that there were millions of bacteria in every square
inch of it in just a few weeks, provided the ocean had the correct
nutrients. And a Von Neumann probe would be far far less picky about what
it considers the "correct nutrients" than a bacteria.

>> OK, there is no disputing matters of taste. But I think you would have
> to agree that not every intelligent being in the galaxy is likely to feel
> about that the same way as you do . as a matter of fact you're
> communicating with such a being right now.
>
>
> *> But only those with the capability to do it count. *


Yes but as I said before, provided I survive the meat grinder singularity,
which I freely admit is problematic, I will have that capacity in a decade
or two. And so will you. But if we don't survive something will, and I
don't believe that every single one of those somethings will have the exact
same likes and dislikes that you do. At least one something is going to
want to try a little cosmic engineering.


> >  *And we're something like the third or fourth dominant species on this
>> planet; dominant in the sense of preventing any other potentially
>> technological species.*
>
>
Sorry, I'm not following you.

   John K ClarkSee what's on my new list at  Extropolis

cme

-- 
You received this message because you are subscribed to the Google Groups 
"Everything List" group.
To unsubscribe from this group and stop receiving emails from it, send an email 
to everything-list+unsubscr...@googlegroups.com.
To view this discussion on the web visit 
https://groups.google.com/d/msgid/everything-list/CAJPayv1YXJKMjxeZ%3DXDGNv6pGpFgJoi4q%3DyKPJyh1Na-Zm9Bew%40mail.gmail.com.


Re: Dyson Spheres and the Fermi Paradox

2024-08-19 Thread John Clark
On Mon, Aug 19, 2024 at 6:17 PM Brent Meeker  wrote:


> *> **if the Milky way has on produced a handful of vNps then it's not so
> implausible that none of have succeeded or even tried.*
>

I don't know how you figure that. It seems ridiculously implausible to me
because it only takes one.


> * > Personally I have no interest in spreading self-reproducing machines
> thru the galaxy*
>

OK, there is no disputing matters of taste. But I think you would have to
agree that not every intelligent being in the galaxy is likely to feel
about that the same way as you do . as a matter of fact you're
communicating with such a being right now.
  John K ClarkSee what's on my new list at  Extropolis

sab


>

-- 
You received this message because you are subscribed to the Google Groups 
"Everything List" group.
To unsubscribe from this group and stop receiving emails from it, send an email 
to everything-list+unsubscr...@googlegroups.com.
To view this discussion on the web visit 
https://groups.google.com/d/msgid/everything-list/CAJPayv00r-%3D14WvTWm1M-4r6MtzZAcuX0%3DC2WUZnj4dLJ8S74A%40mail.gmail.com.


Dyson ​Spheres​ and the Fermi Paradox

2024-08-19 Thread John Clark
Dyson Spheres and the Fermi Paradox
> 

Quotation from the paper:


> *"We discuss the necessity of concepts like Kardeshev Type I/II
> civilizations and Dyson spheres, which would aim to harness vast amounts of
> energy. Even with much larger populations than today, the total energy use
> of human civilization would be orders of magnitude below the threshold for
> causing direct thermal heating or reaching the scale of a Kardashev Type I
> civilization. Any extraterrestrial civilization that likewise achieves
> sustainable population levels may also find a limit on its need to expand,
> which suggests that a galaxy-spanning civilization as imagined in the Fermi
> paradox may not exist. [...]  If human civilization can meet its own energy
> demands with only a modest deployment of solar panels, then this
> expectation might also suggest that concepts like Dyson spheres would be
> rendered unnecessary in other technospheres. These results also have
> implications for the problem known as the Fermi paradox or The Great
> Silence : if extraterrestrial expansion through the galaxy is relatively
> easy, then where are they?"*



That is a great question, it's a pity the paper doesn't provide a great
answer. There are lots of different excuses used to explain the
observational fact that even our largest telescopes see nothing that
appears to have been engineered. However, except for the theory that ET
does not exist, all those excuses have one thing in common, they all assume
there is total (and I do NOT mean near-total) universal agreement among
every single intelligent being in the observable universe that they will
never EVER build a Von Neumann Probe. But If there was just one individual
dissenter then the universe would never look the same.

I think the idea that not one individual in one extraterrestrial
civilization anywhere in the observable universe wants to make a von
Neumann probe is, to put it mildly, implausible. Hell, I'm in the
observable universe and I'd make a Von Neumann Probe if I had the ability.
And, if I survive the Singularity (admittedly a very big IF) then in a
decade or two I will have that ability, and so will you.

I don't believe anybody would put up such a preposterous theory unless they
desperately wanted ET to be real. There is a much simpler idea that doesn't
need that assumption but can explain why neither this galaxy nor any other
galaxy looks like it has been engineered. ET does not exist. The only other
theory that explains the observational facts that is not completely
ridiculous is that it's impossible for any intelligent entity to survive
the Singularity. Personally I think that is unlikely, but FAR more likely
than the idea that nobody, and I do mean nobody, wants to engage in cosmic
engineering.

 John K ClarkSee what's on my new list at  Extropolis

yvx

-- 
You received this message because you are subscribed to the Google Groups 
"Everything List" group.
To unsubscribe from this group and stop receiving emails from it, send an email 
to everything-list+unsubscr...@googlegroups.com.
To view this discussion on the web visit 
https://groups.google.com/d/msgid/everything-list/CAJPayv2NRq%3DZTJ1zhRQ1sde25fo-b%2BSuyyguazig_DQ%2Bcy%3DNNQ%40mail.gmail.com.


Re: Question on the Lorentz Transformation (LT)

2024-08-18 Thread John Clark
On Sun, Aug 18, 2024 at 2:12 AM Alan Grayson  wrote:

*>Suppose a train passes a station traveling at constant velocity .75c and
> a passenger on the train is moving forward at .75c with respect to the car
> in which he resides. What will be the velocity of the passenger with
> respect to the station? I'm pretty sure his velocity is less than c, and
> that this can be determined using the LT, but I forget how it's done,*


The answer is .96c. You could use the formula (v - u) / (1 - vu/c^2) to
calculate this, where v=+.75c because it is moving to the right and u=
-.75c because it is moving to the left,  the opposite direction.
 John K ClarkSee what's on my new list at  Extropolis

o88

-- 
You received this message because you are subscribed to the Google Groups 
"Everything List" group.
To unsubscribe from this group and stop receiving emails from it, send an email 
to everything-list+unsubscr...@googlegroups.com.
To view this discussion on the web visit 
https://groups.google.com/d/msgid/everything-list/CAJPayv14zkUFJzKk77-7L-D9Ya2p_zPGLT322J%2BJgxZ_hK09eA%40mail.gmail.com.


Politics, AI and Quantum Computing

2024-08-14 Thread John Clark
People have been predicting that deep fakes will have an impact on the
election and now, thanks to Elon Musk, it looks like that prediction is
coming true:

Kamala Harris with a gun, Barack Obama stabbing Joe Biden


And after 8 years of study the National Institute of Standards has finalized
Post-Quantum Encryption Standards, they involve Lattice Cryptography:

NIST Releases First 3 Finalized Post-Quantum Encryption Standards


 John K ClarkSee what's on my new list at  Extropolis

lbc

-- 
You received this message because you are subscribed to the Google Groups 
"Everything List" group.
To unsubscribe from this group and stop receiving emails from it, send an email 
to everything-list+unsubscr...@googlegroups.com.
To view this discussion on the web visit 
https://groups.google.com/d/msgid/everything-list/CAJPayv16Qq%2Bog1P7eM5TxC86WrHxNxWPC_oJXQGbbMeU_UMmDw%40mail.gmail.com.


Do Members Of MAGA Deserve The Benefit Of The Doubt?

2024-08-12 Thread John Clark
Do Members Of MAGA Deserve The Benefit Of The Doubt?


John K ClarkSee what's on my new list at  Extropolis

w3m

-- 
You received this message because you are subscribed to the Google Groups 
"Everything List" group.
To unsubscribe from this group and stop receiving emails from it, send an email 
to everything-list+unsubscr...@googlegroups.com.
To view this discussion on the web visit 
https://groups.google.com/d/msgid/everything-list/CAJPayv0dKZ7wfMjEqerpsxBsh_x1cR-cz0MpPwhNogjVryWZSQ%40mail.gmail.com.


Quantum computers are advancing much faster than scientists expected

2024-08-12 Thread John Clark
*The below is a quote from the linked article: *


*"Most people working in the field, however, believe that quantum computers
will be able to solve problems that classical computers can’t solve within
the next 10 years. This is according to a recent survey of 927 people with
associations to the field of quantum computing (researchers, executives,
press, enthusiasts, etc.) conducted by QuEra. Of those surveyed, 74.9%
“expect quantum to be a superior alternative to classical computing for
certain workloads” within the next 10 years.When asked how rapidly the
field of quantum computing was advancing, more than half said either
“faster than I expected” or “much faster than I expected.”  Among
academics, that number rose to 55%."*

*Quantum computers are advancing much faster than scientists expected*


John K ClarkSee what's on my new list at  Extropolis


cqx

-- 
You received this message because you are subscribed to the Google Groups 
"Everything List" group.
To unsubscribe from this group and stop receiving emails from it, send an email 
to everything-list+unsubscr...@googlegroups.com.
To view this discussion on the web visit 
https://groups.google.com/d/msgid/everything-list/CAJPayv1zA%3DKbLVC_6qoCwPyAXokRpJs5eP6r9GuvfbwB5P1jDQ%40mail.gmail.com.


NYTimes.com: Trump Falsely Claims That the Crowds Seen at Harris Rallies Are Fake

2024-08-12 Thread John Clark
Explore this gift article from The New York Times. You can read it for free
without a subscription.

Trump Falsely Claims That the Crowds Seen at Harris Rallies Are Fake

The former president, in a series of social media posts, said that Vice
President Kamala Harris had used A.I. technology to create images of fake
crowds at her events.

https://www.nytimes.com/2024/08/11/us/politics/trump-harris-crowds-ai.html?unlocked_article_code=1.CU4.5IFq.Yho8sXx94FZB&smid=em-share

-- 
You received this message because you are subscribed to the Google Groups 
"Everything List" group.
To unsubscribe from this group and stop receiving emails from it, send an email 
to everything-list+unsubscr...@googlegroups.com.
To view this discussion on the web visit 
https://groups.google.com/d/msgid/everything-list/CAJPayv2gtMtd97seCGUyxJnT%3Deht-XPigC5MsLZZ%3DP6A6i%3DN2Q%40mail.gmail.com.


Cubic millimetre of brain mapped in spectacular detail

2024-08-10 Thread John Clark
This article includes electron microscopic photographs of brain cells that
are the most detailed and beautiful that I have ever seen.

Cubic millimetre of brain mapped in spectacular detail


John K ClarkSee what's on my new list at  Extropolis

tfb

-- 
You received this message because you are subscribed to the Google Groups 
"Everything List" group.
To unsubscribe from this group and stop receiving emails from it, send an email 
to everything-list+unsubscr...@googlegroups.com.
To view this discussion on the web visit 
https://groups.google.com/d/msgid/everything-list/CAJPayv21ZQq9oQJvsUdBiwNrhpypT5K8szEp5vHu6gNfE9hjww%40mail.gmail.com.


NYTimes.com: Yes, Trump Was in a Scary Helicopter Ride. But Not With That Politician.

2024-08-10 Thread John Clark
Explore this gift article from The New York Times. You can read it for free
without a subscription.

Yes, Trump Was in a Scary Helicopter Ride. But Not With That Politician.

There was a helicopter. It did make an emergency landing. But a former
California lawmaker says Donald Trump has mixed up one Black lawmaker for
another.

https://www.nytimes.com/2024/08/10/us/politics/trump-helicopter-crash-willie-brown.html?unlocked_article_code=1.B04.am9P.SPSoqzAYm9-V&smid=em-share

-- 
You received this message because you are subscribed to the Google Groups 
"Everything List" group.
To unsubscribe from this group and stop receiving emails from it, send an email 
to everything-list+unsubscr...@googlegroups.com.
To view this discussion on the web visit 
https://groups.google.com/d/msgid/everything-list/CAJPayv2OM_b57hSCTeRZByvuMsPx27mJ%2BnR0T9XkCs_qexs5HQ%40mail.gmail.com.


NYTimes.com: This Texas Energy Is So Bountiful, They Pay You to Take It Away

2024-08-09 Thread John Clark
Explore this gift article from The New York Times. You can read it for free
without a subscription.

This Texas Energy Is So Bountiful, They Pay You to Take It Away

Natural gas has traded at negative prices for weeks at a time in West
Texas, where pipelines often lack the capacity to get the fuel to places
that need it.

https://www.nytimes.com/2024/08/08/business/energy-environment/natural-gas-negative-prices-texas.html?unlocked_article_code=1.Bk4.Dk0H.qxQ6XCRl5yiZ&smid=em-share

-- 
You received this message because you are subscribed to the Google Groups 
"Everything List" group.
To unsubscribe from this group and stop receiving emails from it, send an email 
to everything-list+unsubscr...@googlegroups.com.
To view this discussion on the web visit 
https://groups.google.com/d/msgid/everything-list/CAJPayv3W%3DQ0HGfrCbYMZefe4Dk4BEVPBzW04K7RGqgU4-ghP9A%40mail.gmail.com.


Project 2025

2024-08-07 Thread John Clark
Dozens of top people in Trump's first administration, and undoubtedly will
be top members of his second administration if the bastard wins the
election, have provided a blueprint of what they intend to do. It's almost
as if after reading Orwell's 1984 they said to themselves, "what a neat idea
, let's make it a reality".  Trump apologists and those who claim there's
no substantive difference between Trump and Harris really need to watch
this:

Project 2025: A Hellish Legal Vision For America


John K ClarkSee what's on my new list at  Extropolis

738

-- 
You received this message because you are subscribed to the Google Groups 
"Everything List" group.
To unsubscribe from this group and stop receiving emails from it, send an email 
to everything-list+unsubscr...@googlegroups.com.
To view this discussion on the web visit 
https://groups.google.com/d/msgid/everything-list/CAJPayv30wcvZFqRp0tyqcYo_5YciBgSU_udxCyt%3DtYZ8%3Deo0TQ%40mail.gmail.com.


Re: [Extropolis] NYTimes.com: JD Vance Just Blurbed a Book Arguing That Progressives Are Subhuman

2024-08-06 Thread John Clark
On Tue, Aug 6, 2024 at 10:12 AM Terren Suydam 
wrote:


> *> Wow. Vance lends a supportive blurb to a book that says that democracy
> is a failure and needs to be replaced by an authoritarian regime. This is
> nakedly un-American. I can't believe anyone who isn't an extremist can
> support that.  But extremism is mainstream on the right now, and this blurb
> is an example of how those on the right are no longer paying any price for
> saying the quiet things out loud, and the overton window for this kind of
> talk is continuing to shift.*
> *And as evidence for how little truth means on the right, a common refrain
> at Trump rallies is that voting to keep the left in power will spell the
> end of democracy. They are in the business of saying whatever people want
> to hear and planting ideas that spread hatred and division, in order to
> attain and stay in power. That exists on the extreme left too, but again,
> it's not mainstream.*
>

I agree with every word you wrote, however you criticize Trump so you
should prepare yourself for a furious attack by Will Steinberg directed at
you; at least that has been my experience whenever I implied that Trump was
somewhat less than perfect.

John K Clark

-- 
You received this message because you are subscribed to the Google Groups 
"Everything List" group.
To unsubscribe from this group and stop receiving emails from it, send an email 
to everything-list+unsubscr...@googlegroups.com.
To view this discussion on the web visit 
https://groups.google.com/d/msgid/everything-list/CAJPayv3d_NYKXnGtYt%3DTVNHnj45PVSsmjTzHnc7YAZfay%3Djhig%40mail.gmail.com.


NYTimes.com: JD Vance Just Blurbed a Book Arguing That Progressives Are Subhuman

2024-08-06 Thread John Clark
Explore this gift article from The New York Times. You can read it for free
without a subscription.

JD Vance Just Blurbed a Book Arguing That Progressives Are Subhuman

A MAGA-world celebration of Francisco Franco and Joseph McCarthy.

https://www.nytimes.com/2024/08/05/opinion/jd-vance-fascism-unhumans.html?unlocked_article_code=1.A04.azyl.iqyleuISpgS4&smid=em-share

-- 
You received this message because you are subscribed to the Google Groups 
"Everything List" group.
To unsubscribe from this group and stop receiving emails from it, send an email 
to everything-list+unsubscr...@googlegroups.com.
To view this discussion on the web visit 
https://groups.google.com/d/msgid/everything-list/CAJPayv2YqB_PQXQLVYshK6O9%2Bd7RS28KZn-MtKLhk4VTC_Cnnw%40mail.gmail.com.


Re: [Extropolis] Re: Scott Aaronson on the November election and some other stuff

2024-08-06 Thread John Clark
On Mon, Aug 5, 2024 at 4:21 PM Keith Henson  wrote:

* >> That's what I'm talking about, your theory can make "predictions"
>> about history, that is to say you can make predictions about things AFTER
>> they happened but not before, and therefore those predictions are of no use
>> whatsoever and it's not a scientific theory.*
>
>
>
> * > Hmm.  Major rain is followed by floods, we know that from history.
> So when we have a majoir rain, what do you expect?*


Anybody can see the obvious connection between rain and a flood, but nobody
can see an obvious connection between lack of food and a war. History tells
us there have always been periods of starvation and there have always been
wars, but if there is a connection between the two it is a very weak one
with lots and lots of exceptions.


> * >> Sometimes shortages occur, and after that wars sometimes happen, and
>> sometimes they do not. In the mid 1970s the US had a severe oil shortage,
>> far far more severe than any shortage we've had since, but it didn't lead
>> to a war or to a Trump-like demagogue gaining power.*
>
>
>
>
>
> * > The evolution that led to such psychological traits happened before
> agriculture when the only shortage of consequence was food.  I don't think
> the oil crisis of the 70s had that much effect on food or the prospects for
> food. *


There was no mass starvation in the USA during the 20th century, so
according to you the USA should've had no wars during the 20th century, but
that is not the case.


> *> there was a food shortage in the US prior to the Civil War. It was not
> recognized at the time, but the historical records show stunting in the
> children of that time.*


Compared with the 20th century, in the 19th century there was a food
shortage in every country in the world resulting in the stunting of
children. And the same thing could be said about the 18th century, and the
17th, and the 16th etc. So no matter what war in history I pick you can
always find a food shortage somewhere to "explain" it. Your theory has no
predictive power because it is so flexible it can be made to explain ANYTHING
and therefore it is impossible to falsify. That means it is not a good
theory.

Worldwide human height has steadily increased over the past 2 centuries


 John K ClarkSee what's on my new list at  Extropolis

whs

-- 
You received this message because you are subscribed to the Google Groups 
"Everything List" group.
To unsubscribe from this group and stop receiving emails from it, send an email 
to everything-list+unsubscr...@googlegroups.com.
To view this discussion on the web visit 
https://groups.google.com/d/msgid/everything-list/CAJPayv3osQnFJRazzbijELMuNDuf0tF83DvigD_L_HGo_isTyQ%40mail.gmail.com.


The best text to picture AI program is free

2024-08-05 Thread John Clark
I think this is pretty remarkable, it's called "Flux" and it's free, and
open source, and it's small enough to run on your home desktop computer if
it has a $1000 graphics accelerator card in it. I'm glad I don't make my
living as a commercial artist!

The best text to picture AI program is free forever


John K Clark

-- 
You received this message because you are subscribed to the Google Groups 
"Everything List" group.
To unsubscribe from this group and stop receiving emails from it, send an email 
to everything-list+unsubscr...@googlegroups.com.
To view this discussion on the web visit 
https://groups.google.com/d/msgid/everything-list/CAJPayv2xQ4PMEn_82%3DnKxFWZtXygg4TEQ%3DUx2TXzVsgNyzKseQ%40mail.gmail.com.


Re: [Extropolis] Re: Scott Aaronson on the November election and some other stuff

2024-08-05 Thread John Clark
On Mon, Aug 5, 2024 at 1:56 PM Keith Henson  wrote:


>
>
> *> If the model is correct, it would be predictive.  As it is, we can look
> at the historical record.  And, indeed, we see that resource shortages or
> perceptions of them preceded wars and related social disruptions. *


That's what I'm talking about, your theory can make "predictions" about
history, that is to say you can make predictions about things AFTER they
happened but not before, and therefore those predictions are of no use
whatsoever and it's not a scientific theory.

Sometimes shortages occur, and after that wars sometimes happen, and
sometimes they do not. In the mid 1970s the US had a severe oil shortage,
far far more severe than any shortage we've had since, but it didn't lead
to a war or to a Trump-like demagogue gaining power. So what was so special
about 2016, and in 2015 did you predict it would be special? What about the
November election, can you predict what the outcome will be? Before Biden
dropped out I thought I could, but now I don't know.


> * >I think it does have predictive power.  It is consistent
> with evolutionary psychology.*


For a psychological theory to be true it needs to be consistent with
Evolution, but it takes more than that because being consistent with
Evolution does not necessarily mean it's true.

John K Clark

-- 
You received this message because you are subscribed to the Google Groups 
"Everything List" group.
To unsubscribe from this group and stop receiving emails from it, send an email 
to everything-list+unsubscr...@googlegroups.com.
To view this discussion on the web visit 
https://groups.google.com/d/msgid/everything-list/CAJPayv2n9QHPaNmPgwdja%3DfNQ7BbjCC5_xMZUGhEvrpr%3DBooBA%40mail.gmail.com.


Re: [Extropolis] Re: Scott Aaronson on the November election and some other stuff

2024-08-05 Thread John Clark
On Sun, Aug 4, 2024 at 8:15 PM Keith Henson  wrote:

*>> On physicist Scott Aaronson's Quantum Computing blog he occasionally
>> changes topics and talks about politics; recently he did so in a dialectic
>> manner about Trump, and it's remarkable how closely his opinions coincide
>> with my own, except that he can express them better.  *
>
> *Never-Trump From Here to Eternity” FAQ
>>  *
>
>
> * > I replied to his screed.*


*Screed? I thought it was logical and concise and covered all the points
that needed to be covered and covered nothing irrelevant. Did Aaronson say
something that was illegal or immoral or just plain wrong? Did he keep
repeating himself, did he say anything that was an exaggeration? If he did
I have not discovered it.*

* > Why is there Trump/MAGA/QAnon interest/agitation/etc now and why is
> it confined to certain areas?  What is different about the current day and
> the past and what is different about those areas?*


*Those are all EXCELLENT questions and your theory can explain all of them,
BUT ONLY AFTER THEY HAVE HAPPENED. Regardless of what turns out to have
occurred your theory can ALWAYS find a way to explain it, therefore it has
zero predictive ability. A good theory such as General Relativity predicted
that when the light of a star passes near the sun it will be deflected by a
certain very specific amount, if it had turned out that it was deflected by
more or less than that precise figure then even Einstein would've had to
admit that General Relativity was wrong because there was no easy way to
change the theory so that it would match the new measurement . You can't
fudge General Relativity because the parts it all fit together like a Swiss
watch. Einstein stuck his neck out but he won. Your theory can explain
anything, which is equivalent to explaining nothing*.

*> I have found this very hard for most people to grasp*


*I understand what you're saying and I'm NOT saying you're wrong, I'm
saying it's not a scientific theory because it has no predictive power. *

John K ClarkSee what's on my new list at  Extropolis

yab





> > Oh and by the way, also on the blog Professor Aaronson informs us that
> just a few days ago the smallest Busy Beaver number that is known to be
> consistent with Zermelo–Fraenkel set theory has been reduced from BB(745)
> to BB(643).
>

fzs

-- 
You received this message because you are subscribed to the Google Groups 
"Everything List" group.
To unsubscribe from this group and stop receiving emails from it, send an email 
to everything-list+unsubscr...@googlegroups.com.
To view this discussion on the web visit 
https://groups.google.com/d/msgid/everything-list/CAJPayv2wy0ZrncCuBkaPgJjxhJhBGArGnybAcks6JAiwDGCx9w%40mail.gmail.com.


Scott Aaronson on the November election and some other stuff

2024-08-04 Thread John Clark
On physicist Scott Aaronson's Quantum Computing blog he occasionally
changes topics and talks about politics; recently he did so in a dialectic
manner about Trump, and it's remarkable how closely his opinions coincide
with my own, except that he can express them better.

*Never-Trump From Here to Eternity” FAQ*


Oh and by the way, also on the blog Professor Aaronson informs us that just
a few days ago the smallest Busy Beaver number that is known to be
consistent with Zermelo–Fraenkel set theory has been reduced from BB(745)
to BB(643).

 John K ClarkSee what's on my new list at  Extropolis

fzs

-- 
You received this message because you are subscribed to the Google Groups 
"Everything List" group.
To unsubscribe from this group and stop receiving emails from it, send an email 
to everything-list+unsubscr...@googlegroups.com.
To view this discussion on the web visit 
https://groups.google.com/d/msgid/everything-list/CAJPayv1%2BqY5JHyreF1Skfa9a-YsxNgGjTARB5SK4kVqzcSSZXQ%40mail.gmail.com.


The Leggett-Garg inequality is violated, so things are not realistic

2024-08-01 Thread John Clark
*Reality says that a macroscopic object exists in one and only one state
regardless of if it has been observed or not. **In 1985 Anthony Leggett and
Anupam Garg published an inequality that MUST be less than or equal to 1 if
reality was true. It's similar to Bell's Inequality but Bell was about the
relationship between two entangled particles, but Leggett-Garg is about if
a microscopic object can be in more than one state at the same instant in
time. *

*In the June 24, 2024 issue of the journal Physical Review Letters,
physicists tested the Leggett-Garg Inequality in an experiment with neutron
beams, and they got a value of 1.20 +- 0.007. That is larger than 1. The
Leggett-Garg inequality is violated. Reality is untrue.*

*Violation of a Leggett-Garg Inequality Using Ideal Negative Measurements
in Neutron Interferometry*


*In their experiment they generated an intense neutron beam and then, using
a perfect silicon crystal, they split it into two beams several centimeters
apart. Then, using another crystal, the two beams are re-combine back in
the one beam and then hit the detector. Each beam is made up of many
millions of neutrons and thus is huge by quantum standards, and there are
two ways the neutrons can travel from the source to the detector.  *

*The lead researcher says "The idea that maybe the neutron is only
traveling on one of the two paths, we just don’t know which one” has thus
been refuted." Mathematically there is simply no way the behavior of those
neutrons can be explained by any conceivable macroscopically realistic
theory. *

*Incidentally, Many Worlds is NOT a realistic theory. *

John K ClarkSee what's on my new list at  Extropolis

bjg

-- 
You received this message because you are subscribed to the Google Groups 
"Everything List" group.
To unsubscribe from this group and stop receiving emails from it, send an email 
to everything-list+unsubscr...@googlegroups.com.
To view this discussion on the web visit 
https://groups.google.com/d/msgid/everything-list/CAJPayv0PV9vcde%2B%2Be6mSsT%2B7KpoUNRdheN6eML1i1BX5E0qHug%40mail.gmail.com.


Re: Are Philosophical Zombies possible?

2024-07-31 Thread John Clark
On Wed, Jul 31, 2024 at 10:49 AM PGC  wrote:

*> I checked the paper again and instead of a response through my phone,
> I'll try to be a bit clearer and leave out the psychology: *
>
> *The misconception is that humans cannot be Turing machines because they
> lack infinite tape. In "On Computable Numbers, With An Application To The
> Entscheidungsproblem," Turing describes Universal Turing Machines (UTMs)
> using finite tables of instructions, with no inherent need for infinite
> resources. Each UTM operates based on a finite set of rules and
> transitions, handling finite inputs and outputs. The notion of an infinite
> tape is often misinterpreted; Turing's idea was that the tape could be
> extended as needed for any computation, ensuring sufficient resources for
> finite tasks rather than literally requiring an infinite resource. Turing's
> formalism re UTM does not involve actual infinities; UTMs are finite
> machines with finite instructions. *
>
> *Turing addresses the ambiguity of functions that may not halt, consistent
> with practical computing where some algorithms may run indefinitely.
> Therefore, the argument that humans cannot be Turing machines due to a lack
> of infinite tape is based on a misinterpretation of Turing's work. Turing's
> detailed description of UTMs involves finite tables, instructions, and
> inputs, making it clear that UTMs are finite in every way. This aligns with
> the practical realities of computation and human cognition, reinforcing the
> idea that human cognitive processes can be viewed as computational within
> Turing's theoretical framework.*
> *I mean, if you approach a UTM with a set of instructions that require the
> full expression of some transcendental number committed to memory in
> decimals, then hopefully the UTM is rich enough in reasoning abilities to
> ask you for your medical history/habits, instead of letting you start
> expressing the full description of your instructions. 😅 Looking at the
> paper, this seems too absurd to even mention there.*
>

Another way to say that is that whenever you observe a Turing Machine that
has stopped (a.k.a. has finished its calculation) you will find it has only
used a finite amount of tape, and whenever you observe a Turing Machine
that is still running you will also observe that it has only used a finite
amount of tape.

John K ClarkSee what's on my new list at  Extropolis

mt0

-- 
You received this message because you are subscribed to the Google Groups 
"Everything List" group.
To unsubscribe from this group and stop receiving emails from it, send an email 
to everything-list+unsubscr...@googlegroups.com.
To view this discussion on the web visit 
https://groups.google.com/d/msgid/everything-list/CAJPayv0WD8LcHDhxkKwpnp6fXKTU%2BN0vEzz9WtG3bS9FFf_Gdw%40mail.gmail.com.


NYTimes.com: Artificial Intelligence Gives Weather Forecasters a New Edge

2024-07-30 Thread John Clark
Explore this gift article from The New York Times. You can read it for free
without a subscription.

Artificial Intelligence Gives Weather Forecasters a New Edge

The brainy machines are predicting global weather patterns with new speed
and precision, doing in minutes and seconds what once took hours.

https://www.nytimes.com/interactive/2024/07/29/science/ai-weather-forecast-hurricane.html?unlocked_article_code=1._E0.dLi6.mwB4Tkx1UuS3&smid=em-share

-- 
You received this message because you are subscribed to the Google Groups 
"Everything List" group.
To unsubscribe from this group and stop receiving emails from it, send an email 
to everything-list+unsubscr...@googlegroups.com.
To view this discussion on the web visit 
https://groups.google.com/d/msgid/everything-list/CAJPayv3DUaTVUb5x-OzE6A_zYz93%2BTusT4DU-OU7xGMU-reQAw%40mail.gmail.com.


That's not me!

2024-07-29 Thread John Clark
*It's not easy having a ridiculously boring and common name like "John
Clark". I was reading an article about Trump giving a speech at a right
wing evangelical Christian convention that said John Clark, a 26 year old
graphic designer from Minneapolis attended the convention and said he was
going to vote for Trump but “I don’t think Trump’s perfect, he’s not Jesus,
so best not to idolize him. As much as he represents conservative values,
he also doesn’t, in a lot of ways. Like, being, let’s say, hateful in some
ways.”*

*I hope I'm a little more eloquent than that, and I'm not 26, I'm not an
evangelical Christian, I'm not a graphic designer, I've never been to
Minneapolis, and I'm not going to vote for Trump; but I do agree with Mr.
Clark that Trump is hateful and it's a good idea to not idolize him.*

John K ClarkSee what's on my new list at  Extropolis
<https://groups.google.com/g/extropolis>
hit

3ez

-- 
You received this message because you are subscribed to the Google Groups 
"Everything List" group.
To unsubscribe from this group and stop receiving emails from it, send an email 
to everything-list+unsubscr...@googlegroups.com.
To view this discussion on the web visit 
https://groups.google.com/d/msgid/everything-list/CAJPayv0ApYW0o%3DEh85t%2BW_9Lhna4_1VLLkZskrLF3mNz6NkpyQ%40mail.gmail.com.


Re: Zuckerberg goes SCORCHED EARTH. Llama 3.1 BREAKS the AGI Industry

2024-07-27 Thread John Clark
*That should be TSMC not TSML.*

*John K Clark*



On Sat, Jul 27, 2024 at 3:54 PM John Clark  wrote:

> On Sat, Jul 27, 2024 at 1:29 PM PGC  wrote:
>
> *" The parallels to past tech bubbles are striking, such as dot-com crash*
>
>
> The two things are not comparable. The invention of the World Wide Web
> was an interesting development, but AI is more profound than the invention
> of fire. There is quite simply nothing in human history they can compare
> with the recent developments in AI and what we will see in the near future,
> and by that I mean in the next 10 years or less, possibly much less.
>
> *> AI is burning through cash at a staggering rate*
>
>
> Yep. That's the problem, if you're a software company and you spend a lot
> of money on AI you MIGHT go bankrupt, but if you're a software company and
> you DON'T spend a lot of money on AI then you're CERTAIN to go bankrupt.
>
>
>>
>> *> AI is, next to impressing us all, also starting to show signs that it
>> might just be the next bubble waiting to burst.*
>
>
> Thanks to Zuckerberg and the rise of open source, AI software companies
> like Open AI and Anthropic may go belly up, but not hardware companies like
> ASML, TSML and Nvidia. Open source or closed, **somebody** (maybe private
> companies, maybe government. maybe the military) is certain to be
> developing and running AI programs that require a gargantuan amount of
> computation, and they will need hardware to do that, so companies that are
> involved in that should do well. Although TSML might go out of business
> but not because AI fizzles out but because of a Chinese invasion of
> Taiwan.
>
>  > *This year alone, investors are expected to pour $60 billion into AI
>> development—enough to create 12,000 products the size of OpenAI's ChatGPT.*
>
>
> Most of those products will flop but a few will grow exponentially.
>
> * > Do we need that many?*
>
>
> We do unless you can figure out a sure fire way to separate the good from
> the bad.
>
> * > Jim Covello recently remarked [...]*
>
>
> Do you really think Jim Covello knows what's going on?
>
>  John K ClarkSee what's on my new list at  Extropolis
> <https://groups.google.com/g/extropolis>
> 9$z
>
> z
>

-- 
You received this message because you are subscribed to the Google Groups 
"Everything List" group.
To unsubscribe from this group and stop receiving emails from it, send an email 
to everything-list+unsubscr...@googlegroups.com.
To view this discussion on the web visit 
https://groups.google.com/d/msgid/everything-list/CAJPayv0V0k_TnXCiqYWuaywx-tf4kCe5TCfQS8fG2aJPUn8n3w%40mail.gmail.com.


Re: Zuckerberg goes SCORCHED EARTH. Llama 3.1 BREAKS the AGI Industry

2024-07-27 Thread John Clark
On Sat, Jul 27, 2024 at 1:29 PM PGC  wrote:

*" The parallels to past tech bubbles are striking, such as dot-com crash*


The two things are not comparable. The invention of the World Wide Web was
an interesting development, but AI is more profound than the invention of
fire. There is quite simply nothing in human history they can compare with
the recent developments in AI and what we will see in the near future, and
by that I mean in the next 10 years or less, possibly much less.

*> AI is burning through cash at a staggering rate*


Yep. That's the problem, if you're a software company and you spend a lot
of money on AI you MIGHT go bankrupt, but if you're a software company and
you DON'T spend a lot of money on AI then you're CERTAIN to go bankrupt.


>
> *> AI is, next to impressing us all, also starting to show signs that it
> might just be the next bubble waiting to burst.*


Thanks to Zuckerberg and the rise of open source, AI software companies
like Open AI and Anthropic may go belly up, but not hardware companies like
ASML, TSML and Nvidia. Open source or closed, **somebody** (maybe private
companies, maybe government. maybe the military) is certain to be
developing and running AI programs that require a gargantuan amount of
computation, and they will need hardware to do that, so companies that are
involved in that should do well. Although TSML might go out of business but not
because AI fizzles out but because of a Chinese invasion of Taiwan.

 > *This year alone, investors are expected to pour $60 billion into AI
> development—enough to create 12,000 products the size of OpenAI's ChatGPT.*


Most of those products will flop but a few will grow exponentially.

* > Do we need that many?*


We do unless you can figure out a sure fire way to separate the good from
the bad.

* > Jim Covello recently remarked [...]*


Do you really think Jim Covello knows what's going on?

 John K ClarkSee what's on my new list at  Extropolis

9$z

z

-- 
You received this message because you are subscribed to the Google Groups 
"Everything List" group.
To unsubscribe from this group and stop receiving emails from it, send an email 
to everything-list+unsubscr...@googlegroups.com.
To view this discussion on the web visit 
https://groups.google.com/d/msgid/everything-list/CAJPayv1ppHQmrydFwfE7GF6AtHgT%2BEezo_3%3DrgKA1FYbkYWvyg%40mail.gmail.com.


Re: [Extropolis] Fwd: Cryonics Chances

2024-07-25 Thread John Clark
On Thu, Jul 25, 2024 at 3:53 PM Keith Henson  wrote:

>>
>> *I also agree with Hanson chemical fixation is a very promising
>> alternative to liquid nitrogen that deserves much more emphasis:*
>>
>
> * > **I don't think it makes a lot of difference. The difference between
> zero and what it costs to keep a neuro patient stored is about $200/year. *
>

*Being cheaper is nice but that's not chemical fixation's primary virtue.
Electron microscopic pictures of brains preserved by chemical fixation are
clearer, have fewer artifacts, and give us much more information about
brain structure than brains preserved with liquid nitrogen .*

*> Being recovered from suspension probably takes nanotechnology. *
>

*Yes absolutely, if it wasn't for nanotechnology I wouldn't be interested
in Cryonics.  Back in 2013 on the Extropian List I responded to an article
that was in cryonics magazine by Aschwin de Wolf trying to make the point
that cryonic suspension was better than Chemical fixation, *
*  http://www.alcor.org/Library/html/chemopreservation.html
 . This is what I
said: *

* > For example, the expensive and extremely toxic chemical osmium
> tetroxide is routinely used for stabilization of lipids in preparation for
> electron microscopy.*
>

*If something as dangerous to handle and expensive as osmium tetroxide were
needed then that could be a show stopper for chemical fixation being an
alternative to cryonics, but I don't see why it would be needed. Osmium
tetroxide is primarily not a stabilizer but a stain, it works well as a
contrast agent in electron microscopes because heavy metals like Osmium
scatter lots of electrons. However, how information is extracted from my 3
pounds of frozen or chemically fixed brain is not my problem, it is the
problem of beings who live in an age of advanced Nanotechnology. My only
concern is that the information remains intact inside that 3 pounds of grey
goo, I don't know exactly how it will be extracted but I doubt it will be
by electron microscopes.  *

* > Unlike the cryobiologist, the chemical fixation researcher cannot
> reverse fixation and test for viability.*
>

*I don't understand what is meant by that. or what edge Cryonics has over
fixation because of it. Viability just means it works, and so far neither
Cryonics nor fixation has brought anybody back and I don't think anybody
will until advanced nanotechnology is developed. It almost sounds like
there is supposed to be some advantage in using the same atoms in the
reawakened being as in the old one, but I can't imagine what that advantage
could be.*

*> The cryobiologist does not have to confine himself to this fate because
> he can attempt to measure viability in the brain*
>

*Obviously during revival at every step you'd like to know if you're doing
it right and are on the right track, but again I don't see why cryonics
would be better at this than fixation.*

*> or even the whole organism.*
>

*Preserving any part of the body with either method except for the brain
seems completely pointless to me.*

*> Let us assume, for the sake of the argument, that the chemopreservation
> advocate has identified a number of fixatives (and other treatments) that
> are sufficient for complete ultrastructural preservation of the brain. The
> next question is going to be: how stable will chemopreservation be over
> time? This is a very important point for the technical feasibility of
> chemopreservation.*
>

*Yes, that is an important point. With Cryonics, unless we're talking about
millions of years and as long as things remain cold (but will it?),  pretty
much all the damage that is going to be done has been done by the time the
brain reaches liquid nitrogen temperatures. And I'm not worried about
damage caused during thawing because that won't be done with existing
technology, assuming it's even thawed at all and it probably won't be; the
information will probably be read out by disassembling the brain from the
outside in while it remains in solid form.*

*I don't know if chemical fixation would remain as stable over the
centuries as freezing, my hunch is that cryonics has a small edge over
fixation in this regard but I could be dead wrong, maybe it's a big edge.
And I don't want to be dead. *


> * > It is not only necessary to demonstrate that all chemicals can be
> introduced by perfusion fixation without perfusion artifacts*


*Both methods are imperfect so it is only necessary to demonstrate that
fixation produces fewer artifacts than Cryonics or that the artifacts
produced are easier to identify as artifacts to make it the superior
technology.  *

*> In my opinion, the prospect of autolysis is much worse because when
> biomolecules break up into their constitutive parts, and go into solution,*
>

*True, but if fixation is done correctly there won't be any fluid for
things to move in.*

*> there is a risk that essential parts of the brain will not be fixed, as
> a result 

NYTimes.com: Move Over, Mathematicians, Here Comes AlphaProof

2024-07-25 Thread John Clark
Explore this gift article from The New York Times. You can read it for free
without a subscription.

Move Over, Mathematicians, Here Comes AlphaProof

A.I. is getting good at math — and might soon make a worthy collaborator
for humans.

https://www.nytimes.com/2024/07/25/science/ai-math-alphaproof-deepmind.html?unlocked_article_code=1.900.M5Vr.2uC_-qzQM0JM&smid=em-share

-- 
You received this message because you are subscribed to the Google Groups 
"Everything List" group.
To unsubscribe from this group and stop receiving emails from it, send an email 
to everything-list+unsubscr...@googlegroups.com.
To view this discussion on the web visit 
https://groups.google.com/d/msgid/everything-list/CAJPayv1gCkMiL0aBRk52K3eQixnwPd55Dhqu83HORYwngRA-mQ%40mail.gmail.com.


Freedom of the press and Donald Trump

2024-07-25 Thread John Clark
We have just seen a preview of what we can expect if Trump wins the
election and somebody says something that he doesn't like. Donald Trump has
sued ABC News and George Stephanopoulos for defamaton because he said had
been found liable for raping  E. Jean Carroll. Yesterday Republican
appointed Florida judge Cecilia Altonaga refused to dismiss the case as
frivolous and allowed it to continue despite the fact that the judge in the
trial that found Trump guilty said that Trump had been found guilty of
rape * “as
many people commonly understand the word"* though not as it is narrowly
defined by New York state law.

Judge won’t dismiss Trump’s defamation suit against ABC News and George
Stephanopoulos


John K ClarkSee what's on my new list at  Extropolis

fpu

-- 
You received this message because you are subscribed to the Google Groups 
"Everything List" group.
To unsubscribe from this group and stop receiving emails from it, send an email 
to everything-list+unsubscr...@googlegroups.com.
To view this discussion on the web visit 
https://groups.google.com/d/msgid/everything-list/CAJPayv0-XUxsLaEmuOabJ7W_5z_GAZd8AZWjV5moS_LQRSNc-w%40mail.gmail.com.


Fwd: Cryonics Chances

2024-07-25 Thread John Clark
Robin Hanson believes, as do I, that:

*"People who “die” today could live again in the future, perhaps forever,
as brain emulations (= uploads, ems), if enough info were saved today about
their brains. (And of course if civilization doesn’t die, if someone in the
future cares enough to bother) This is probably enough brain info: the
spatial shape and location of each brain cell, including the long skinny
parts that stick out to touch other cells, and two dozen chemical densities
(at the skinny part scale) to help identify cell and connection types.
Actually, it is probably enough to just get 95% of the connections right,
and a half dozen chemical densities."*

I also agree with Hanson chemical fixation is a very promising alternative
to liquid nitrogen that deserves much more emphasis:

*"**If we used chemical bonds to fix proteins in place, we could store
cryonics patients at much closer to room temperature, and then they’d not
need to be stored or managed by centralized orgs that could fail. Full-body
patients could be buried as mummies in caskets, while brain-only patients
could be squirreled away in small 1300cc containers. They might be stored
in secret locations, perhaps in isolated permafrost, perhaps with pointers
to those locations stored and protected cryptographically, only to be
revealed to an advanced civ later. Yes, fixation makes restarting biology
harder, but that isn’t much of an obstacle to brain emulation, which seems
to me pretty sure to be the first feasible revival tech." *
-- Forwarded message -
From: Robin Hanson from Overcoming Bias 
Date: Wed, Jul 24, 2024 at 9:08 PM
Subject: Cryonics Chances
To: 


Attending an event on cryonics this last weekend tempts me to revisit the
topic. And given the crazy tiny number of folks who have signed up for it
(~4K), compared to the vast numbers of people (e.g., ~30% of my followers,
& ~10% of my undergrads) who say that they think the product makes sense
for them, the big cryonics marketing puzzle is: why aren’t far more people
customers? The unexplained ratio (~5% of 8B / 4K) is roughly a factor of
~10,000!
͏     ­͏     ­͏     ­͏     ­͏     ­͏     ­͏     ­͏     ­͏     ­͏     ­͏
­͏     ­͏     ­͏     ­͏     ­͏     ­͏     ­͏     ­͏     ­͏     ­͏     ­͏
  ­͏     ­͏     ­͏     ­͏     ­͏     ­͏     ­͏     ­͏     ­͏     ­͏     ­͏
    ­͏     ­͏     ­͏     ­͏     ­͏     ­͏     ­͏     ­͏     ­͏     ­͏
­͏     ­͏     ­͏     ­͏     ­͏     ­͏     ­͏     ­͏     ­͏     ­͏     ­͏
  ­͏     ­͏     ­͏     ­͏     ­͏     ­͏     ­͏     ­͏     ­͏     ­͏     ­͏
    ­͏     ­͏     ­͏     ­͏     ­͏     ­͏     ­͏     ­͏     ­͏     ­͏
­͏     ­͏     ­͏     ­͏     ­͏     ­͏     ­͏     ­͏     ­͏     ­͏     ­͏
  ­͏     ­͏     ­͏     ­͏     ­͏     ­͏     ­͏     ­͏     ­͏     ­͏     ­͏
    ­͏     ­͏     ­͏     ­͏     ­͏     ­͏     ­͏     ­͏     ­͏     ­͏
­͏     ­͏     ­͏     ­͏     ­͏     ­͏     ­͏     ­͏     ­͏     ­͏     ­͏
  ­͏     ­͏     ­͏     ­͏     ­͏     ­͏     ­͏     ­͏     ­͏     ­͏     ­͏
    ­͏     ­͏     ­͏     ­͏     ­͏     ­͏     ­͏     ­͏     ­͏     ­͏
­͏     ­͏     ­͏     ­͏     ­͏     ­͏     ­͏     ­͏     ­͏     ­͏     ­͏
  ­͏     ­͏     ­͏     ­͏     ­͏     ­͏     ­͏     ­͏     ­͏     ­͏     ­͏
    ­͏     ­͏     ­͏     ­͏     ­͏     ­͏     ­͏     ­͏     ­͏     ­͏
­͏     ­͏     ­͏     ­͏     ­͏     ­͏     ­͏     ­͏     ­͏     ­͏     ­͏
  ­͏     ­͏     ­͏     ­͏     ­͏     ­͏     ­͏     ­͏     ­͏     ­͏     ­͏
    ­͏     ­͏     ­͏     ­͏     ­͏     ­͏     ­͏     ­

Cryonics Chances


Robin Hanson 
Jul 25






Zuckerberg goes SCORCHED EARTH. Llama 3.1 BREAKS the AGI Industry

2024-07-24 Thread John Clark
It looks like Mark Zuckerberg is going to do to AGI what Linus Torvalds did
to Unix, and I am delighted. It also looks like during the next few years
the big money will not be made by software companies but by hardware
companies such as ASML, Nvidia and TSML. And lack of powerful hardware is
the one disadvantage that China has.

Zuckerberg goes SCORCHED EARTH. Llama 3.1 BREAKS the AGI Industry


John K ClarkSee what's on my new list at  Extropolis

zma

-- 
You received this message because you are subscribed to the Google Groups 
"Everything List" group.
To unsubscribe from this group and stop receiving emails from it, send an email 
to everything-list+unsubscr...@googlegroups.com.
To view this discussion on the web visit 
https://groups.google.com/d/msgid/everything-list/CAJPayv1HvHAqeqJg5SWwwqoDowPeRigXCtEXH8UyO5HjTDn16A%40mail.gmail.com.


Zuckerberg is democratizing the singularity

2024-07-24 Thread John Clark
 Stuart LaForge wrote this for my Extropolis list, I thought people over
here might be interested:
==

On Wed, Jul 24, 2024 at 3:40 AM Stuart LaForge  wrote:

I cannot keep track of all the dominoes that will fall because of what
> Mark Zuckerberg did today. If AI is the New World that we are exploring,
> then Mark Zuckerberg just burned our ships like Hernan Cortez. His
> company Meta has developed and trained Llama 3.1 which one of the best
> performing LLMs in the entire market and today he just made it open
> source.  In less than two hours I had the 8 billion parameter version of
> Llama 3.1 up and running on my Windows 11 laptop. Eliezer Yudkowsky is
> probably shitting his pants right now because the genie is all the way
> out of the bottle. But I am more optimistic than I have been since Open
> AI was actually open source. Some greedy men tried to monopolize AI for
> their own power, but Zuckerberg just gave it to the people. In my
> estimation, he has made it possible for freedom and democracy into the
> Singularity and beyond.
>
> If you want run your own local copy of Llama 3.1 (~ 5 GB of hard drive)
> you can either download it from the meta.com site and compile it which
> is a pain. Or if you have have Windows and want a smooth and easy
> automatic install go to ollama.com and install Ollama (which helps you
> install many different AI models) and follow the instructions.
>
> Stuart LaForge

-- 
You received this message because you are subscribed to the Google Groups 
"Everything List" group.
To unsubscribe from this group and stop receiving emails from it, send an email 
to everything-list+unsubscr...@googlegroups.com.
To view this discussion on the web visit 
https://groups.google.com/d/msgid/everything-list/CAJPayv0e4VFNv0YBJOEba0PBkmKM5S2sZFOj0Bm7-oSj28QMig%40mail.gmail.com.


NYTimes.com: Trump’s Favorite Lies, Puffery and Flights of Fancy, Up Against the Data

2024-07-24 Thread John Clark
Explore this gift article from The New York Times. You can read it for free
without a subscription.

Trump’s Favorite Lies, Puffery and Flights of Fancy, Up Against the Data

A demonstration of the many ways the ex-president bends the truth, in
charts.

https://www.nytimes.com/2024/07/24/opinion/trump-lies-charts-data.html?unlocked_article_code=1.9k0.vVRQ.aoWuk75cy7if&smid=em-share

-- 
You received this message because you are subscribed to the Google Groups 
"Everything List" group.
To unsubscribe from this group and stop receiving emails from it, send an email 
to everything-list+unsubscr...@googlegroups.com.
To view this discussion on the web visit 
https://groups.google.com/d/msgid/everything-list/CAJPayv2g9_RXX96OwP3vknFxD257HgODk0L5g9hY5oTNb3m%2BKQ%40mail.gmail.com.


LLaMA 3.1 is out and it will change everything

2024-07-23 Thread John Clark
I think this is a super important day, it's certainly Eliezer Yudkowsky's
nightmare scenario, nobody is going to be able to put the toothpaste back
into the tube now. Meta has just released the AI program LLaMA 3.1 and it's
just as good as the best that has come out of OpenAI even though it only
has 405 billion parameters and GPT4 has 1.6 trillion; Meta made a more
efficient program that can get more out of less. But what's really
revolutionary is that it's completely open source, and not just the code
but for the all important weights as well.  They've also released a much
smaller 8 billion parameter model that is small enough to run on your cell
phone yet is damn good,  it would be considered the smartest AI program in
the world just a couple of years ago.

This news kind of puts things into perspective for me, it makes the current
state of American politics look pretty trivial.

*LLaMA 405b is here! Open-source is now FRONTIER!*


John K ClarkSee what's on my new list at  Extropolis

ptq

-- 
You received this message because you are subscribed to the Google Groups 
"Everything List" group.
To unsubscribe from this group and stop receiving emails from it, send an email 
to everything-list+unsubscr...@googlegroups.com.
To view this discussion on the web visit 
https://groups.google.com/d/msgid/everything-list/CAJPayv2SnqpacT4am56Q0txwLy1_MPZ4oY4goN_ihjOWkRTCUg%40mail.gmail.com.


Re: Biden will not seek reelection; endorses Harris

2024-07-23 Thread John Clark
On Tue, Jul 23, 2024 at 4:49 AM Brent Meeker  wrote:

* > I don't think there's any real alternative to Harris.*
>

It's starting to look like you're right.

* >For Vice President she needs to have a white male Senator, such as Mark
> Kelly of AZ*
>

Mark Kelly would be an interesting choice but it doesn't have to be a
Senator, more governors have gone on to become president than senators, and
governors have executive experience but senators do not.  And I don't see
anything wrong with an all female ticket, we've certainly had plenty of all
male tickets and I'm not exactly thrilled with the results.

John K ClarkSee what's on my new list at  Extropolis

4v7

-- 
You received this message because you are subscribed to the Google Groups 
"Everything List" group.
To unsubscribe from this group and stop receiving emails from it, send an email 
to everything-list+unsubscr...@googlegroups.com.
To view this discussion on the web visit 
https://groups.google.com/d/msgid/everything-list/CAJPayv0poFowiA1z%2BgtCfUOSDWxV4f8RDp4osq8cD8q8H8eXvw%40mail.gmail.com.


Re: Biden will not seek reelection; endorses Harris

2024-07-22 Thread John Clark
On Mon, Jul 22, 2024 at 10:54 AM PGC  wrote:

*> should the Democrats pivot towards a new, dynamic figure that embodies a
> break from the past, or is Kamala Harris the best option they have, merely
> offering an anti-Trump stance without substantial innovation? If so, who
> could that candidate be, *


If they don't pick Kamala Harris a good pick would be Michigan governor
Gretchen Whitmer. She has twice won the race for governor in the must-have
swing state of Michigan, and won by a 10 point margin both times.
Objectively she has the knack of being able to get things done, and
subjectively the people of Michigan like what she has done. And she's
likable and looks good on television. If they don't pick her for president
she would be an ideal choice for Harris's vice president.

John K Clark

>
>

-- 
You received this message because you are subscribed to the Google Groups 
"Everything List" group.
To unsubscribe from this group and stop receiving emails from it, send an email 
to everything-list+unsubscr...@googlegroups.com.
To view this discussion on the web visit 
https://groups.google.com/d/msgid/everything-list/CAJPayv0AA0QysphfFwMzbLyGmat4E_Cucj%2BD%2B0D9Ow%3DC1o7BiA%40mail.gmail.com.


Re: Biden will not seek reelection; endorses Harris

2024-07-22 Thread John Clark
On Mon, Jul 22, 2024 at 10:54 AM PGC  wrote:

*> Can a seasoned insider like Harris convincingly articulate a fresh,
> positive vision for the future? *


Probably not because that would be difficult, but I think she might be able
to do something that was far easier, articulate the nightmarish Orwellian
future we can expect if Trump is in the oval office again.  And that's all
she needs to do to win.

John K Clark





>

-- 
You received this message because you are subscribed to the Google Groups 
"Everything List" group.
To unsubscribe from this group and stop receiving emails from it, send an email 
to everything-list+unsubscr...@googlegroups.com.
To view this discussion on the web visit 
https://groups.google.com/d/msgid/everything-list/CAJPayv1PggGDwFK_TcNC8XE_My1RVL-pPk%3DcdC24nXL8AEiXSw%40mail.gmail.com.


Biden will not seek reelection; endorses Harris

2024-07-21 Thread John Clark
At last  glimmer of hope!

Biden will not seek reelection; endorses Harris


*“It has been the greatest honor of my life to serve as your President,”
Biden wrote. “And while it has been my intention to seek reelection, I
believe it is in the best interest of my party and the country for me to
stand down and to focus solely on fulfilling my duties as President for the
remainder of my term.”*

John K Clark

-- 
You received this message because you are subscribed to the Google Groups 
"Everything List" group.
To unsubscribe from this group and stop receiving emails from it, send an email 
to everything-list+unsubscr...@googlegroups.com.
To view this discussion on the web visit 
https://groups.google.com/d/msgid/everything-list/CAJPayv1gKouSrFggBpGa%3DY9wS19i7TV0EPqnAdzTVKPkz9b3mA%40mail.gmail.com.


Should the Democrats nominate a Republican?

2024-07-21 Thread John Clark
In today's New York Times Aaron Sorkin, the guy who wrote the TV series
"The West Wing" had an interesting idea that I had not considered before.
It's worth thinking about :

*"**At their convention next month, the Democrats should nominate Mitt
Romney. Nominating Mr. Romney would be putting our money where our mouth
is: a clear and powerful demonstration that this election isn’t about what
our elections are usually about, but about stopping a deranged man from
taking power.  Does Mr. Romney support abortion rights? No. Does he want to
aggressively raise the minimum wage, bolster public education, strengthen
unions, expand transgender rights and enact progressive tax reform?
Probably not. But is he a cartoon thug who did nothing but watch TV while
the mob he assembled beat and used Tasers on police officers? No. The
choice is between Donald Trump and not-Trump, and the not-Trump candidate
needs only one qualification: to win enough votes from a cross section of
Americans to close off the former president’s Electoral College path back
to power.  And Mr. Romney could make the case that the Democrats are
putting country before party in ways that the MAGA movement will not, and
announce his bipartisan cabinet picks at the convention as well."*

John K ClarkSee what's on my new list at  Extropolis

iwt

-- 
You received this message because you are subscribed to the Google Groups 
"Everything List" group.
To unsubscribe from this group and stop receiving emails from it, send an email 
to everything-list+unsubscr...@googlegroups.com.
To view this discussion on the web visit 
https://groups.google.com/d/msgid/everything-list/CAJPayv3fZom4xCNekzmEiqMvr_77Ye4_g7PENf2jOuFwFCmoKg%40mail.gmail.com.


Quantum Computers

2024-07-20 Thread John Clark
*Back in 2018 Raz and Tal proved something important, they did not prove
that a quantum computer could solve all nondeterministic polynomial time
problems in polynomial time, but they did prove that even if P=NP, and even
if we had a algorithm that could solve NP problems on a conventional
computer in polynomial time, there would STILL be a class of problems a
conventional computer couldn’t solve efficiently, but a quantum computer
could.  A conventional computer couldn't even efficiently verify that a
solution was correct, much less find it. This newly discovered class of
very exotic problems (they involve the distribution of random numbers) may
be of fundamental interest in themselves or they may be interesting for no
reason other than that a conventional computer can’t solve them but a
quantum computer can. Even if the problems turn out to be useless I think
this discovery is important because this is the first time anybody has
proven that there is at least one thing that a quantum computer is
fundamentally better at than can a conventional computer. *

*The following is a quote from the Raz and Tal paper:*



*"Can polynomial-time quantum algorithms be simulated by classical
algorithms in the polynomial-time hierarchy? In this paper, we show that in
the black-box model (also known as query-complexity or decision-tree
complexity), the answer is negative"*

Oracle Separation of BQP and PH ContactAdd CommentRSS-Feed


John K ClarkSee what's on my new list at  Extropolis

cuq

-- 
You received this message because you are subscribed to the Google Groups 
"Everything List" group.
To unsubscribe from this group and stop receiving emails from it, send an email 
to everything-list+unsubscr...@googlegroups.com.
To view this discussion on the web visit 
https://groups.google.com/d/msgid/everything-list/CAJPayv2Nr7%3DMNsWwAxCLRvpzofDO2Wt-MZAzaLwKS8tHUY8%2BFw%40mail.gmail.com.


Re: Are Philosophical Zombies possible?

2024-07-20 Thread John Clark
On Fri, Jul 19, 2024 at 10:49 PM Russell Standish 
wrote:

* > It's a moot point about whether a human can be considered a
> universal Turing machine*


*I don't think it's a moot point! If a Universal Turing Machine can emulate
a human being (and I see no reason why it could not) then in my humble
opinion that's just about the most interesting and important fact about
existence imaginable. At least imaginable by me. *

*> a human's finite lifetime is a problem*


I don't see why. Any Turing Machine that has stopped (a.k.a. successfully
finished its calculation) has only used a FINITE amount of tape. And the
same thing could be said about any Turing Machine that is still working on
its problem.


> > *there may well be limits to the amount of computation physically
> possible in the universe, depending on the universe's geometry*
>

Irrelevant. If true then that fact would affect a human being just as much
as it would affect a Turing Machine.

 John K ClarkSee what's on my new list at  Extropolis

tio

-- 
You received this message because you are subscribed to the Google Groups 
"Everything List" group.
To unsubscribe from this group and stop receiving emails from it, send an email 
to everything-list+unsubscr...@googlegroups.com.
To view this discussion on the web visit 
https://groups.google.com/d/msgid/everything-list/CAJPayv2rJ8uf0iK%3DpLO8xSCurjDEVO-TXeeiR7nWJWur6Rh88w%40mail.gmail.com.


  1   2   3   4   5   6   7   8   9   10   >